Вы находитесь на странице: 1из 107

134

TIME GENERATED COMPOSITIONS YIELD

April

r e c u r s i o n r e l a t i o n (6) yields the other set of a l t e r n a t e Fibonacci n u m b e r s a s the sequence of cumulative suras, the total p a r t i c l e count.
5.

CONCLUDING REMARKS

One is d i r e c t e d to advanced p r o b l e m H-5Q D e c e m b e r 1964, F i b onacci Q u a r t e r l y , for the partitioning i n t e r p r e t a t i o n of the i n t e g e r


the model for

n of

<p(t) - kt.

Suppose one defines two s e t s of Morgan-Voyce polynomials


b Q (x) = 1, b } (x) = 1 + x; B Q (x) = 1, B ^ x ) = 2 + x

both sets satisfying


(7)

n+2

( x ) = (X + 2 ) P

n+l(x) " Pn(x)j

It is e a s y to e s t a b l i s h that
P (k) = A = k B , (k)
n
n
n-1
T (k) = An + A, + . . . + A = b (k)
n
0
1
n
n

Thus for k = 1, we again find B ,(1) = F n


and b (1) = F~ ( 1 . See

n-1
2n
n
2n+l
c o r r e c t e d p r o b l e m B-26 with solution by Douglas L i n d i n t h e E l e m e n t a r y
P r o b l e m Section of this i s s u e , w h e r e the binomial coefficient r e l a t i o n
mentioned in the note of Section 3 is shown.

A future p a p e r by Prof.

M. N. S. Swamy dealing extensively with M o r g a n - Voyce polynomials will


a p p e a r in an e a r l y i s s u e of the Fibonacci Q u a r t e r l y .
Acknowledgment:

The author is completely indebted to Dr. V. E.

Hoggatt, J r . , for bringing to his attention the t h e o r e m and its proof.


Additional r e f e r e n c e s to work along the lines of g e n e r a t e d c o m p o s i tions - some of which yield n u m b e r s with Fibonacci p r o p e r t i e s w i l l
be found in the r e f e r e n c e s at the end of this p a p e r . (See note, page 94)
REFERENCES
H. Winthrop "A T h e o r y of B e h a v i o r a l Diffusion" A contribution to the
M a t h e m a t i c a l Biology of Social P h e n o m e n a .

Unpublished t h e s i s s u b -

m i t t e d to the Faculty of The New School for Social R e s e a r c h , 1953.


H. Winthrop,

"Open P r o b l e m s of I n t e r e s t in Applied M a t h e m a t i c s , "

M a t h e m a t i c s Magazine, 1964, Vol. 37, pp. 112-118.


H. Winthrop,

"The Analysis of T i m e - F l o w Equivalents in Finite Dif-

f e r e n c e Equations Governing Growth" (In p r e p a r a t i o n ) .


xxxxxxxxxxxxxxx

MYSTERY PUZZLER AND PH!


MARVIN H. HOLT
Wayzata, Minnesota

A p r o b l e m p r o p o s e d by P r o f e s s o r Hoggatt is as follows:

Does

t h e r e exist a p a i r of t r i a n g l e s which have five of t h e i r six p a r t s equal


but which a r e not c o n g r u e n t ?

(Here the six p a r t s a r e the t h r e e sides

and the t h r e e a n g l e s . ) The initial impulsive a n s w e r is no J The p r o b lem a l s o a p p e a r s in | 1 | as well as in the MATH LOG.
I have taken some time to work on the problem you suggested.
I think you will a g r e e that the solution I have is i n t e r e s t i n g .
lem,

One p r o b -

as you have stated it, is posed in a high school g e o m e t r y text

entitled, " G e o m e t r y " b y M o i s e and Downs, published by Addison Wesley


Company, (page 369)*
In t h e i r solution key, they gave one possible p a i r of t r i a n g l e s
that work:

I d i s c o v e r e d this after I solved the p r o b l e m myself.

But the above

solution does not do j u s t i c e to the p r o b l e m at all, since m y old friend


r is r e a l l y the key to the solution.

Note: Golden Mean = 96 = r in what

follows.
I attacked the p r o b l e m as follows: F i r s t , the five congruent p a r t s
cannot contain all t h r e e s i d e s ,
gruent,

since the t r i a n g l e s would then be con-

T h e r e f o r e , the five p a r t s m u s t be t h r e e angles and two sides

which m e a n s that the two t r i a n g l e s a r e s i m i l a r .

But, the two sides

cannot be in c o r r e s p o n d i n g o r d e r , or the t r i a n g l e s would be congruent


e i t h e r by ASA or SAS. So, the situation m u s t be one of two p o s s i b i l i t i e s
as I have sketched below:

(My s k e t c h e s a r e not to s c a l e . )
135

136

MYSTERY P U Z Z L E R AND PHI

April

triangle 2

triangle 1
possibility 1

possibility 2
In both c a s e s , by using r e l a t i o n s h i p s from s i m i l a r t r i a n g l e s , it follows
that |:=
or b = ka and c = kb = k a from p o s s i b i l i t y 2 and ~ = 3 or
2
b d
b = ka and d = kb = k a from p o s s i b i l i t y 1.
So, the t h r e e sides of the t r i a n g l e m u s t be t h r e e consecutive
2
m e m b e r s of a g e o m e t r i c s e r i e s : a, ak, ak , w h e r e k is a p r o p o r t i o n ality constant and k > 0 and k ^ 1. If k = 1, the t r i a n g l e s would both
be e q u i l a t e r a l and thus congruent. T h e r e f o r e , k / 1.
F r o m m y p r e v i o u s a r t i c l e on the Golden Section (Pentagon, Spring
1964) I worked out two p r o b l e m s on right t r i a n g l e s w h e r e the sides
f o r m e d a g e o m e t r i c p r o g r e s s i o n and the c o n s t a n t s t u r n e d out to b e / ^ "
and y , So, I knew of two m o r e situations w h e r e the o r i g i n a l p r o b l e m
could be solved.

Then I began to c o n s i d e r v a r i o u s other values of k

and I began to wonder what values of "kM will work. In other w o r d s , for
what values of k will the n u m b e r s

a, ak, and ak

be sides of a t r i -

a n g l e . Once we know t h i s , then another t r i a n g l e with s i d e s ^ , a, ak


k
2
3
or ak, ak , ak w i l l h a v e five p a r t s congruent but the t r i a n g l e s would
not be congruent.
I n o r d e r f o r a, ak and ak to be sides of a t r i a n g l e , t h r e e s t a t e m e n t s m u s t be t r u e :

1965

MYSTERY P U Z Z L E R AND PHI

137

These a r e i n s t a n c e s of the s t r i c t . t r i a n g l e inequality.


1.
a + ak > a k 2
(a + b > c)
2.

a + ak > ak

3.

ak + ak

[a > 0,

(a + c > b)
(b + c > a)

> a

k^l]

k > 0,

For Case 1, consider k > 1


k > 1>k

(a)

t h e r e f o r e , a + ak
(b)

if

> k

> ak (condition 2 above)


k > l>k + 1 > 1>k

t h e r e f o r e , ak
(c)

> k ->1 + k

+ k >1

+ ak > a (condition 3 above)

k >1

show

(condition 1 above)

a + ak > ak

This p a r t r e v o l v e s around the p r o b l e m of finding out when 1 + k > k ,


2
or, graphically: F o r what x > 1 will 1 + x = y be above y = x ?

p_^-

Jf Y =
2

1 +x

y = x
2

Solving this p r o b l e m p r o d u c e s the r e s u l t that


k <

or_

k < r .

2
So, if 1 < k < r then the n u m b e r s a, ak, ak a r e the sides of the
a
2
3
t r i a n g l e that can be m a t c h e d with -- , a, ak or ak, ak , ak to solve
the o r i g i n a l p r o b l e m . (Incidentally: 1 < v ^ < r . So this fits i n h e r e . )
F o r Case 2, c o n s i d e r k < 1

138

(b)

MYSTERY P U Z Z L E R AND PHI


z
z
if k < 1->k < k ~ > k < k + 1 T h e r e f o r e ak < ak + a
(condition 1)
if k < 1 > 1 + k > 1 >a + ak > ak (condition Z)

(c)

Now, if k < 1 show ak + ak

(a)

April

> a.

This i s ,

essentially,finding

what values of k m a k e k + k > 1.


Again, graphically, for what x < 1 will the p a r a b o l a
be above the line y = 1 ?

y =x +x

pX7p
m

y=l

X=1

-1 + /5~
Solving this p r o b l e m p r o d u c e s the r e s u l t that k >
~
. If you
will follow this closely, -1 +^ / 5 is the additive i n v e r s e of the conjugate
of ^ .

(i. e, j r =

Therefore,

and its additive i n v e r s e is

the conjugate of
I

/"cT

14-

-=

. ) So, if

_L

r is

/C,

< k < 1 the p r o b -

-1 + v/5
/l
lem is again solved. (Again,
< 1, so m y second p r o b l e m
fits h e r e . )
T h e r e f o r e , the complete solution can_be s u m m e d up as follows,
1 + \/5
- I + /5
if k is a n u m b e r such that 1 < k <
^
= r or
^
< k < 1.
Then the t h r e e s e t s of t r i a n g l e s with sides -r- , a, ak or a, ak, ak
or ak, a k 2 , or a k 3 can be used to produce two t r i a n g l e s with five
p a r t s equal and the t r i a n g l e s t h e m s e l v e s not congruent.
So, t h e r e a r e an infinite n u m b e r of p a i r s of t r i a n g l e s that solve
this p r o b l e m and once again, r p r o v e s to be an i n t e r e s t i n g n u m b e r
and a key to the solution of i n t e r e s t i n g problems,,

1.

REFERENCES
Moise and Downs, G e o m e t r y , Addison-Wesley, p. 369.

xxxxxxxxxxxxxxx

LADDER NETWORK ANALYSIS USING POLYNOMIALS


JOSEPH ARKIN
Spring Valley, New York

In this p a p e r we develop some ideas with the r e c u r r i n g s e r i e s


(1)

= k, B , + k^B OJ
B n = 1, (k. and k 0 / 0) ,
n
1 n-1
2 n-2
0
1
2 ' '
and show a r e l a t i o n s h i p between this sequence and the simple network
of r e s i s t o r s known as a Ladder-network.
The l a d d e r - n e t w o r k in F i g u r e 1 is an i m p o r t a n t network in c o m munication s y s t e m s .

The m - L

sections in c a s c a d e that m a k e up this

network can be c h a r a c t e r i z e d by defining:


(2)

a) the attenuation (input voltage/output voltage)


b) the output impedance

= z ,

c) the input impedance

e1

R,

= A,

R,

R,

"1

-O-t*

R~

>

R_

R.

* - .

Figure 1

A r e s u l t obtained by applying Kirchhoff's and Ohm's Laws to


l a d d e r - n e t w o r k s with m = 1, 2, 3, . . . , R, = 'R~k-, was tabulated with
the r e s u l t s in Table 1, w h e r e setting k, = 1, R- = 1 ohm, the network
in F i g u r e 1 was analyzed by inspection [1] .
139

140

ApriL

LADDER NETWORK ANALYSIS


m

0
Rn

(k^l)

VMR2

(k^+3^+1)

(kx+l)R2
k x + 3 k +1 \ R

~k+Z~

k^+3k1+l\R2

'^+5^+6^+1 \ R

(^+5^+6^+1)

k^+4kx+3y

k^+4k1+3

Table 1
We o b s e r v e t h a t t h e
z

m
n

(C

nth

r o w i n T a b l e 1, m a y be w r i t t e n
A

C9
2n

2n-2/y2n-l)R2

(C

2iA2n-l)R2

where,
(3)
x
'

a)

b)

yr

= k

1/2
;
C , + C 9,
1
n-1
n-2

k, '

n - 1 + yn - 2 '

It t h e n r e m a i n s t o s o l v e f o r

Cn
0
y

1/2

= IA;

and C
i n (3), t o be a b l e t o a n a l y z e
n
n
( F i g u r e 1) by i n s p e c t i o n f o r a n y v a l u e of k (k ^ 0), w h e r e R = 1 o h m .
So t h a t , i n (1), w e l e t
(4)

y
J

a)

w =

(k L + (k* + 4 k 2 ) l / 2 ) / 2

b)

(k x - ( k 2

+4k2)l/2)/2

w h e r e it i s e v i d e n t ,
c)

k,

d)

k~

W + V

and
= - wv

T h e n , c o m b i n i n g (c) a n d (d) w i t h (1), l e a d s t o


(5)

((w

B
B

, - wv(w-v)B
)/(w-v)
n-l
n - L9
o o
2 2
= ((w ~v )B 0 - w v (vw - v )B
o)/(w-v)
n-2
' n-3"

- v )B

= ((w

- v )(w+v) - w v ( w "

- v

,
i

)B0)/(w-v)

1965

USING P O L Y N O M I A L S

and we have

,_
n+1

(6)

141

n+1

= --a

w - v
1/2
W h e r e , i n (1) w e r e p l a c e k, w i t h k, '
and
i n g t h i s r e s u l t w i t h (3) a n d (6), l e a d s t o
(k;/2
(7)

a)

and

with

1, a n d c o m b i n -

k1+4)i/2)n+1-(k!/2-(k1+4)1/2)n+1

'

1/2 n + l
((k +4) ' ) 2 n

. /,,
= ^(k^/k}^

b) y n
(8)

k9

=0(k1),
l

Theorem.
T h e a t t e n u a t i o n ( i n p u t v o l t a g e / o u t p u t v o l t a g e = A) of

m-L

tions in c a s c a d e in a l a d d e r - n e t w o r k is given by
2m-2
A2

c ((-c7

)/c9

7)

^
r
2m-l"
2m-2 '
r=0
T h e p r o o f of t h e t h e o r e m r e s t s on t h e f o l l o w i n g
(9)

Lemma,
The power

series
n
(-l)

is always a square, where

Brxr

r =0
is defined i n . ( l ) .

P r o o f of l e m m a ,
Let
n
1 = (1-kjX - k2x2)( 1

(10)

Brxr)

r=0
t h e n , b y c o m p a r i n g c o e f f i c i e n t s a n d b y (1), w e h a v e
- ( B k. + B , k 9 )
n 1
n-.l 2 '
X =
_
n 2

,, . .
(11)
and r e p l a c i n g
(12)
x
'

x with

l-klX-k0x2
1 2

(-B

>i)/(B

?)

in

-B

,.
n+1
= - _ n 2

2
^-kj^-k?*1 ^

l e a d s

= x( B 2 k + B B ^ . k , - B 2 x 1 ) ,// (v B 2 k 9 )
n 2
n n+1 1
n+l
n 2'

B y (4, d) a n d (6) it i s e a s i l y v e r i f i e d

to

sec-

142

LADDER NETWORK ANALYSIS

(13)

B2 - B ,.B . = (-k-)n
n
n+1 n-1
2

so that
(14)
x
'

April

B 2 k + B B . . k . - B 2
= x( - l ) n k ^ + 1
n 2
n n+1 1
n+1
'
2

Then, r e p l a c i n g the n u m e r a t o r in (12) by the r e s u l t in (14) leads to


1-k x - k ? x 2

(15)

= ((-l)nk*)/B2

so that (10) m a y be w r i t t e n a s
n
n

(16)

(-l) B

B xr
r

r=0
which c o m p l e t e s the proof of the l e m m a .
(17) The proof of the t h e o r e m is i m m e d i a t e , when in (11) and (16), we
. _ .
__
__
r e p l a c e n with 21X1-23, k, with k , ' , k ? with 1, and combine the
r e s u l t with (7, a) and the values of the attenuation in Table 1.

REFERENCES
1.

a) S. L. Basin, "The A p p e a r a n c e of Fibonacci N u m b e r s and the


Q M a t r i x in E l e c t r i c a l Network Theory, " Math Mag. , 36(1963)
pp. 84-97 0
b) S. L. Basin,

"The Fibonacci Sequence as it A p p e a r s in Na-

t u r e , " Fibonacci Q u a r t e r l y , 1(1963) pp. 5 4 - 5 5 .


The author e x p r e s s e s his gratitude and thanks to P r o f e s s o r L.
Carlitz,

Duke University;

Professor

V. E. Hoggatt, J r . ,

San J o s e

State College; and the r e f e r e e .


XXXXXXXXXXXXXXX

REQUEST
The F i b o n a c c i Bibliographical R e s e a r c h Center d e s i r e s that any
r e a d e r finding a Fibonacci r e f e r e n c e , send a c a r d giving the r e f e r e n c e
and a brief d e s c r i p t i o n of the c o n t e n t s . P l e a s e f o r w a r d all such inf o r m a t i o n to:
Fibonacci Bibliographical R e s e a r c h C e n t e r ,
Mathematics Department,
San J o s e State College,
San J o s e , California

CONCERNING LATTICE PATHS AND FIBONACCI NUMBERS


DOUGLAS R. STOCKS, JR.
Arlington State College, Arlington, Texas

R s E. Greenwood [1] h a s investigated plane Lattice paths from


(0, 0) to (n, n) and has found a r e l a t i o n s h i p between the n u m b e r of paths
in a c e r t a i n r e s t r i c t e d s u b c l a s s of such paths and the Fibonacci s e quence.

Considering such paths and using a method of e n u m e r a t i o n

different from that used by Greenwood, an unusual r e p r e s e n t a t i o n of


F i b o n a c c i ' s sequence is suggested.
The paths c o n s i d e r e d h e r e a r e c o m p r i s e d of steps of t h r e e types:
(i) h o r i z o n t a l from (x, y) to (x + 1, y); (ii) v e r t i c a l from (x, y) to (x, y + 1);
and (iii) diagonal from (x, y) to (x + 1, y + 1).

H5

H4

H3

H2

HL

Figure 1
In the i n t e r e s t of s i m p l i c i t y of r e p r e s e n t a t i o n , we will h e r e cons i d e r the paths from

H. to V., for each positive i n t e g e r i.

that the n u m b e r of paths from


(0, 0) to (i, i).
paths from

However,

i n s t e a d of c o n s i d e r i n g the total n u m b e r of

H. to V. as was done by Greenwood,

the n u m b e r of paths from


any of the paths from
n u m b e r of paths from
H. to V..

Note

H. to V. is the n u m b e r of paths from

H. to V. which do not contain as subpaths

H. to V., for

H. . ,

we will count only

to V. ,

j < i.

This n u m b e r plus the

is the total n u m b e r of paths from

The u s e of this counting device suggest the


143

144

CONCERNING LATTICE PATHS

ApriL

Theorem:
Let

iD

= i

= 3
D

L where
+2

D-l
=

denotes the g r e a t e s t i n t e g e r functioi

V2

D-l
+ 3

D-2

(2n) D = ( 2 n ) D - 2 + ( 2 n - l ) D - 2
(2n+l)D = ( 2 n + l ) D ^ + ( 2 n ) D - 1
with the r e s t r i c t i o n that k n = 0 if k > D.
D, let

F o r each positive i n t e g e r

f(D) =

S kD
k=l

The sequence {f(D) ] D = 1, 2, 3, . . . } is the Fibonacci s e q u e n c e .


The proof is d i r e c t and is t h e r e f o r e omitted.
The g e o m e t r i c i n t e r p r e t a t i o n of the n u m b e r s

and f(D) m e n -

tioned in the t h e o r e m is i n t e r e s t i n g . However, before c o n s i d e r i n g this


i n t e r p r e t a t i o n it is n e c e s s a r y to define a s e c t i o n of a path.
p u r p o s e we will now c o n s i d e r a path a s the point set to which p

F o r this
belongs

if and only if for s o m e step ((x, y), (u, v)) of the path, p belongs to the
line i n t e r v a l whose end points a r e (x, y) and (u, v). A section of a path
is a line i n t e r v a l which is a s u b s e t of the path and which is not a subset
of any other line i n t e r v a l each of whose points is a point of the path.
The above mentioned g e o m e t r i c i n t e r p r e t a t i o n follows:
nition f(l) = 1.

F o r each positive i n t e g e r

set of paths from


thepathsfrom

Ln.

L~ denote the

to V n which do not contain a s subpaths any of

H. to V., for

longing to the set


Ln

Hn

D > 2, let

By defi-

j < D. f(D) is the n u m b e r of paths b e -

is the n u m b e r of paths in the subset

X of

such that x belongs to X if and only if x contains a s s u b s e t s

exactly k diagonal s e c t i o n s .

1965

AND FIBONACCI NUMBERS


F i g u r e 2 p o r t r a y s the five paths which belong to

145

L-.

In F i g u r e

2a a p p e a r s the one path of L- which contains only one diagonal s e c tion ( 1 - = 1). The two paths of L- which contain exactly two diagonal
sections a p p e a r in F i g u r e 2b (2- = 2)*

In F i g u r e 2c the two paths of

L c which contain exactly t h r e e diagonal sections a r e shown ( 3 r = 2).


o
5
It is noted that 4_ '= 5 r = 0.

F i g . 2a
f(5)

Fig. 2b

Fig. 2c

25=2

35=2

1 +2+2+0+0 = 5
Figure 2

REFERENCES
1.

R. E. Greenwood,

"Lattice

Paths

and Fibonacci

Numbers,

The Fibonacci Q u a r t e r l y , Vol. 2, No. 1, pp. 13-14.


XXXXXXXXXXXXXXX

NOTICE TO A L L SUBSCRIBERS] I I
P l e a s e notify the Managing Editor AT ONCE of any a d d r e s s change.
The P o s t Office D e p a r t m e n t , r a t h e r than forwarding m a g a z i n e s m a i l e d
t h i r d c l a s s , sends them d i r e c t l y to the d e a d - l e t t e r office.

Unless the

a d d r e s s e e specifically r e q u e s t s the Fibonacci Q u a r t e r l y be forwarded


at f i r s t c l a s s r a t e s to the new a d d r e s s , he will not r e c e i v e it.
will usually cost about 30 cents for f i r s t - c l a s s p o s t a g e . )

(This

If p o s s i b l e ,

p l e a s e notify us AT LEAST THREE WEEKS PRIOR to publication d a t e s ;


F e b r u a r y 15, A p r i l 15, October 15, and D e c e m b e r 15.

REPLY TO EXPLORING FIBONACCI MAGIC SQUARES*


JOHN L . BROWN, JR.
Pennsylvania State University, State College, Pennsylvania

Problem.
F o r n > 2, s h o w t h a t t h e r e do n o t e x i s t a n y n x n m a g i c
s q u a r e s w i t h d i s t i n c t e n t r i e s c h o s e n f r o m t h e s e t of F i b o n a c c i n u m b e r s , u , = 1, u , = 2, u , n = u . , + u f o r n > I .
1
2
n + 2 n+1 n
Proof.

T r i v i a l for
If a n n x n

n = 2.

magic

Fibonacci entries,

s q u a r e e x i s t e d for

some

n > 3 with distinct

then the r e q u i r e m e n t that the f i r s t t h r e e c o l u m n s

a d d to t h e s a m e n u m b e r w o u l d y i e l d t h e e q u a l i t i e s :
(*) F . + F .
1 2

+. . . + F.
n

= F . + F . +. . . + F . = F 1
+ F . +. . . + F .
.
J
J
J
l
2
n
1
2
n

S i n c e t h e e n t r i e s a r e d i s t i n c t , w e m a y a s s u m e w i t h o u t l o s s of g e n e r ality that F . > F . > . . . > F . , F . > F . > . . . > F . a n d


X
F

k, >
1

k >
2

2
F

' >

k
n

Noting t h a t the c o l u m n s c o n t a i n no c o m m o n e l e m e n t s , and by r e a r r a n g e m e n t if n e c e s s a r y , w e a s s u m e

F. > F. > F

generality; thus,
Now

, again without losing

F. > F
+ 2.
1
k
l
l
F.

+ F.

i,

i9

+ . . . + F.

> F . > F,

i ^ i , -

,-

k, +2

while
k
F

k,
1

+F

k +--2 ?

+F

2 1

T h i s c o n t r a d i c t s t h e e q u a l i t y p o s t u l a t e d i n (*), a n d w e c o n c l u d e no m a g i c
s q u a r e s in distinct F i b o n a c c i n u m b e r s a r e p o s s i b l e .

The Fibonacci Q u a r t e r l y ,

O c t o b e r 1964, P a g e 216.

xxxxxxxxxxxxxxx
146

THE FIBONACCI NUMBER F u WHERE u IS NOT AN INTEGER


ERICHALSEY
Redlands, California

INTRODUCTION
Fibonacci n u m b e r s , like f a c t o r i a l s , a r e not n a t u r a l l y defined for
any values except integer v a l u e s . However the gamma, function extends
the concept of f a c t o r i a l to n u m b e r s that a r e not i n t e g e r s .
that (l/Z)I = ^/JT/2.
F

for any i n t e g e r

rational number

Let

n but which will f u r t h e r m o r e

give

f(x, y) = x ^

= 1 (Definitions (1) hold for all n c N)

Let
rdjL

(read "n c a r d i n a l " )

= 2

k^

= 2

k=l
This gives the c a r d i n a l n u m b e r s 1, 2, 3,
Let

2
n$

1 = n

k=l

1
(read "n t r i a n g u l a r " )

2 k$
k=l

= 2 k
k=l

This gives the t r i a n g u l a r n u m b e r s 1, 3, 6, 10,


Let

...

n
n$3

(read "n t e t r a h e d r a l " )

k^2

k=l
This gives the t e t r a h e d r a l n u m b e r s 1, 4, 10, 20, . .
In g e n e r a l , let
n
n$

give

for any
and d e -

w h e r e x and y need not be i n t e g e r s 0

DEFINITIONS
n$

u. The a r t i c l e also defines a quantity n $

velops a function
(1)

Thus we find

This a r t i c l e develops a function which will

(read "n d e l t a - s l a s h m") = 2


k$
k=l
147

148

April
WHERE u
u
This gives a figurate n u m b e r s e r i e s which can be a s s i g n e d to the
THE FIBONACCI NUMBER F

m - d i m e n s i o n a l analog of the t e t r a h e d r o n (which is the 3 - d i m e n s i o n a l


analog of the t r i a n g l e , etc. ).
Let us c o n s t r u c t an a r r a y

( a. .), w h e r e we a s s i g n to each

a. .

an a p p r o p r i a t e coefficient of P a s c a l ' s t r i a n g l e .

(a.

.)

i,3

10

15

10

20

35

15

35

70

It is c l e a r that in this a r r a n g e m e n t the usual rule for forming P a s c a l ' s


t r i a n g l e is just
a. . = a. . - + a. . .

(2)

But a c o m p a r i s o n of this rule with the definitions (1) shows that P a s c a l ' s
t r i a n g l e can be w r i t t e n :

where

a1

i^2

2#

2jLl

2 /

3^

3JL1

3fi2

n(L

n$

i<

...

2 ^

3Ar

a. . = iA

a. . = a. 1..
iJ
J*
(3)

1/

F r o m the s y m m e t r y of P a s c a l ' s t r i a n g l e ,
Therefore
i^"1

= j^1"1;

nfi = (m+l)^ 1 1 " 1

P a s c a l ' s t r i a n g l e is a well-known g e n e r a t o r of Fibonacci n u m b e r s in


the way shown in the following d i a g r a m .

1965

IS NOT AN INTEGER

149

/ / / / /
1 / 1 / 1

1 = 1 = F,

1 = 1 = F_

10

15

1+1

10

20

35

1+2

15

35

70

1+3+1 = 5 = F r

We can apply the s a m e c o u r s e to our a b s t r a c t e d P a s c a l ' s t r i a n g l e .


/ / /
^0

3^ /3tLl

3^

F 3 = 3<X + l ^ 1

F 4 = 4^ + Zfk1

It is c l e a r that, if we keep forming Fibonacci n u m b e r s from P a s c a l ' s


t r i a n g l e in this way, F = n ^ + (n-2)$ + (n-4)$ + . . . + ( n - 2 m ) $ , or

(4)

(n-2k) K

k=0

w h e r e we r e q u i r e that m b e a n i n t e g e r and that 0 < n - 2 m . < 2, or in


other w o r d s that n / 2 - 1 <_ m < n / 2 .
(5) T h e o r e m 1
Proof:
E(n).

Now let us prove

^m _ ( n + m - 1 ]

It is sufficient to p e r f o r m induction on n.
Then if n = 1, E(l) s t a t e s

/ n + m - l \ _ / l + m - l \ _ mj_
\
m /
\ m /
ml

_ .

Let the t h e o r e m be

150

THE FIBONACCI NUMBER F

B u t b y d e f i n i t i o n (1), ( m + l ) $
t i o n (3)

1^

= 1

for

= 1 for any

WHERE u
(m+1) t N .

m = 0, 1, 2, 3, . . .

let us a s s u m e that, for a r b i t r a r y


n

Mm

and

m i N, E(n)

April
Then by e q u a -

E(l)

is t r u e .

is t r u e .

Then

Now

/n+m-l\
= (
m
)

F r o m t h e d e f i n i t i o n s (1) it c a n be s e e n t h a t
. vm - 1
. *m - 1
Mm - 1 .
1$
+ 2^
+ . . . + nfi

^m
= n^k

T h e r e f o r e t h e i n d u c t i o n h y p o t h e s i s c a n be r e s t a t e d

if1

(6)
Add

+ Z ^ ' 1 + . . . + ( ^ m I 2 ) = (n+-1)

) t o b o t h s i d e s of e q u a t i o n (6) t o o b t a i n

lA""1 iZfi"-1 + . . . +("l m i 2 ) + (i"i')

(7)

The right-hand

s i d e of e q u a t i o n (7) i s

for c o m b i n a t i o n s ,

) by t h e s t a n d a r d i d e n t i t y

so we have

or

m 1

m 2

I^ 111 - 1 + z^ - + ... +( ^ i )+(

(n+

^!!r" 2 )

((n+l)+m-l\
which is

E(n+1).

Therefore

E(n)

implies

E(n+1)

and T h e o r e m 1 is

t r u e by m a t h e m a t i c a l i n d u c t i o n .
Now l e t u s p r o v e
1
(8)

Theorem 2

n^

= [(n+m) f

xn'l(l

-x)mdx J"1

0
Proof:

r(n) = (n-l)I
B ( m , n)

( g a m m a function)

= B(n, m )

= -1 . ' ~}--

(beta function)

1965

IS NOT AN INTEGER

151

Therefore
1
_
r(m-fn)
B(m, n) " r(m)r(n)

'

and
1
, n-m+1)
(n

r(n+2)
r(m+l) r(n-m+l)

4 ^ v , = (n+l) C)

(n+l)!
ml (n-m)!

ml (n-m):

Then
(9)
w /

(n)
\m/

= 7-xrvoT-4i
m = [(n+l)B(m+l,n-m+l)] "l
(n+l )B(m + l, n-m+1)
Lx
'J

We can now substitute the right-hand side of equation (5) into equation
(9) to obtain
n m

= (n+_1) = [(n+m^m+^n)]-1

where

-,
B(m+l,n)

= B(n 5 m + 1) = J

x111 (1-x) m dx

0
Therefore
1
n

x11"l (1 - x ^ d x ] " 1

= [(n+m) J

0
Both equations (5) and (8) a s s e r t that n $

= (m+l) " . Some inter-

esting special cases of equation (5) are

JD

^
vl

=
=

/n-l\

(o)

/ n\

(n-l)I

k^nr
n!

( l ) = (n-l)ill

=l
=

'

and

_/

n + 1

( n + 1 )i

_ (n)(n+l)

k=l.
Now we can put equation (8) into equation (4) to obtain

152

THE FIBONACCI NUMBER F

WHERE u

April

u
1

(10)

Fn -

[(n-k) J

k=0

2k-F
x n -
^l-x^dx]"1

w h e r e m is an i n t e g e r , n / 2 - 1 < m < n / 2 .

But w h e r e a s equations

(4) and (5) have m e a n i n g only for i n t e g e r a r g u m e n t s , equations (8) and


(10) can be used to find

x$/

and F

w h e r e x, y, and u a r e any

rational numbers.
In p a r t i c u l a r
m
(11)

F,

1 [<u-k) J
k=0

x^^^d-x^dx]"

w h e r e m is an i n t e g e r , u / 2 - 1 < m < u / 2 e The equation (11), and


the definite i n t e g r a l in it, a r e e a s i l y p r o g r a m m e d for solution on a
digital c o m p u t e r . A few values of F
follow.

4. 1000000
4. 2000000
4. 3000000
4. 4000000
4. 5000000
4. 6000000
4. 7000000
4. 8000000
4. 9000000
5. 0000000
5..1000000
5* 2000000
5. 3000000
5. 4000000
5. 5000000
5,.6000000
5. 7000000
5, 8000000
58.9000000
6. 0000000

3. 1550000
3. 3200000
3.4950000
3. 6800000
3. 8750000
4. 0800000
4. 2950000
4. 5200000
4. 7550000
5. 0000000
5. 2550000
5. 5200000
5. 7950000
6. 0800000
6. 3750000
6. 6800000
6.,9950000
7. 3200000
7, 6550000
8. 0000000
xxxxxxxxxxxxxxx

0. 1
0.2

1.0
1.0
1.0
1. 1
1.2
2.0
2. 1

4.0

3.0

ELEMENTARY PROBLEMS AND SOLUTIONS


Edited by A.P. HILLMAN
University of Santa Clara, Santa Clara, California

Send all c o m m u n i c a t i o n s r e g a r d i n g E l e m e n t a r y P r o b l e m s and


Solutions to P r o f e s s o r A. P . Hillman, M a t h e m a t i c s D e p a r t m e n t ,
v e r s i t y of Santa C l a r a , Santa C l a r a , California.

Uni-

Any p r o b l e m believed

to be new in the a r e a of r e c u r r e n t sequences and any new a p p r o a c h e s


to existing p r o b l e m s will be w e l c o m e d .

The p r o p o s e r should submit

each p r o b l e m with solution in legible form, p r e f e r a b l y typed in double


spacing with n a m e and a d d r e s s of the p r o p o s e r as a headinge
Solutions to p r o b l e m s should be submitted on s e p a r a t e s h e e t s in
the f o r m a t used below within two months of publication.
B-64

Proposed by Verner E. Hoggatt, Jr., San Jose State College, San Jose,

California

Show that

L L ,, = L~ , , + ( - 1 ) . w h e r e L is the n-th
n n+1
2n+l
n
n u m b e r defined byy L, = 1, L 0 = 3, and L l 0 = L M + L .
1
2
n+^
n+1 n
B-65

Proposed by Verner E. Hoggatt, Jr., San Jose State College, San Jose,

Let u

and v be s e q u e n c e s satisfying
n
n
v . T+CV ,, +dv =0 w h e r e a, b, c, and d
n+Z
n+1
n
A
% 7
(E 2 +aE+b)(E 2 +cE+d) = E 4 + p E +qE + r E + s . Show
y
7

B-66

j / i p y j-2+qy

n+4

r/

n+3

n/

.o

n+2

+r

y
7

.i

n+l

+ s

Lucas

California

u , J a u ,,+bu =0 and
n+2
n+1
n
a r e constants and let
that y =u +v

satisfies

Proposed by D.G. Mead, University of Santa Clara, Santa Clara,

California

Find constants p, q, r, and s such that


yn+4+pyn+3+qyn+2+ryn+1+syn = o
is a 4th o r d e r r e c u r s i o n r e l a t i o n f o r the t e r m - b y - t e r m p r o d u c t s y =u v
3
r
J
n n n
of solutions of u l 0 - u M - u = 0 and v , 0 - 2 v ,, -v = 0.
n+2 n+1 n
n+2
n+1 n
B-67

Proposed by D.G. Mead, Uni versity of Santa Clara, Santa Clara,


B

and

California

Find the sum 1' 1 +1 * 2+2 5+3' 12+. . . +F G , w h e r e F ^ = F ,. +F


n n
n+Z n+1 n
Gn+2=2Gn+1+Gn.
153

154

ELEMENTARY PROBLEMS AND SOLUTIONS

B-68

April

Proposed by Walter W. Horner, Pittsburgh, Pennsylvania

Find e x p r e s s i o n s in t e r m s of Fibonacci n u m b e r s which will gene r a t e i n t e g e r s for the d i m e n s i o n s and diagonal of a r e c t a n g u l a r p a r a l l e l opiped, i. e. , solutions of
2,, 2 , 2
,2
a +b +c = d
B-69

Proposed by Werner E. Hoggatt, Jr., San Jose State College, San Jose,

California

Solve the s y s t e m of simultaneous equations:


xF
xF

where

is the n - t h

,, +yF = x +y
J
n+1 J n

n+2+yFn+l

x2+2xy

Fibonacci n u m b e r .
SOLUTIONS

CHEBYSHEV POLYNOMIALS
B-27

Proposed by D.C. Cross, Exeter,

Corrected
Polynomials

England

and r e s t a t e d from

Vol. 1, No. 4:

The Chebyshev

P (x) a r e defined by P (x) = c o s ( n A r c c o s x).

Letting

(>

f = A r c c o s x, we have
:os <f> - x = P (x),
cos (2$) = 2cos 0

- 1 = 2x

- 1 = P?(x),

cos (3<f>) = 4cos 0 - 3cos <j> = 4x

- 3x = P~(x),

cos (40) = 8 c o s 4 0 - 8 c o s 2 0 + 1 = 8 x 4 - 8x 2 + 1 = P 4 ( x ) ,
It is well known that
P

n + 2< x > =

2xP

n+l<X> " Pn(x>

Show that
P x(x) =
n

m
I
j=0

B. x n ' 2 j
jn

'

etc.

155

ELEMENTARY PROBLEMS AND SOLUTIONS

155

where

m = [n/2 J ,
the g r e a t e s t i n t e g e r not exceeding n / 2 , and
(1) B
= 2n_1
'
on
(2) B . , , x l = 2 B . . .
- B.
.
j+l,n+l
j+l,n
j,n-l
(3) I f S = | B I + | B . | + . . . + | B
I,
n
' on'
'In1
' mn'

then S

10

n+2

Solution by Douglas Lind, University of Virginia, Charlottesville,

= 2S ,. + S
n+1
n

Va.

By De M o i v r e ' s T h e o r e m ,
(cos <f> + i sin </>) -

cos n 0 + i sin rub

Letting x = cos <fi, and expanding the left side,


/2
= (x + i vl - x )

cosn0+isinn0
.

= 2 (-l) j / 2 (Jx n - j (l - x V / 2
j=o
We equate r e a l p a r t s , noting that only the even t e r m s of the sum a r e
real,

[n/2]
cos n< = P j x ) =

( - l ) k ( 2 ^) x n " 2 k ( l - x 2 )

k=0
We m a y p r o v e from this (cf. F o r m u l a (22), p. 185, Higher T r a n s c e n d tal F u n c t i o n s ,
Numbers,

Vol. 2 by E r d e l y i et al; R. G. Buschman,

Chebyshev P o l y n o m i a l s ,

G e n e r a l i z a t i o n s and

Equations, " Fibonacci Q u a r t e r l y , Vol. 1, No. 4, p. 2) that


(*)

B.
J, n

11 \ - l /

\1L-J-Xf.

H (n-Zj):

F r o m t h i s , we have

(1)

o, n

= 2n~l

"Fibonacci
Difference

156

ELEMENTARY PROBLEMS AND SOLUTIONS

April

It is a l s o e a s y to show from (*) that


(2)

B. ,,
,,
j+l,n+l

= 2 B. ,,
- B.
.
J+l,n
J, n-1

Now (*) i m p l i e s
B.
= (-1) J | B .
J,n
' j,n'
so that (2) b e c o m e s
( " D j + 1 |1 B .j ++1l , n ++ Jl J = 2 ( - l ) j + 1 |'B .j++Tl , n JI + ( - D J + 1 |'B .j , n -,l

B.,,

,,

= 2

B.,T

B.

..

j + 1 , n+1 '
' j+1, n 1
j , n-1
Summing both sides for j to I =J, we have
(3)
v
'

S ,. = 2 S + S .
n+1
n
n-1
Also solved by the proposer.

A SPECIAL
B-52

CASE

Proposed by Venter E. Hoggatt, Jr., San Jose State College, San Jose,

California

F ^ - F 2 = (x - l ) n + 1 , w h e r e F is the n - t h F i b n-20 n+2


n
n
onacci n u m b e r , defined by F , = F~ = 1 and F 0 = F , + F .
3
1
2
n+2
n+1
n
Showthat

Solution by John L. Broivn, Jr., Pennsylvania

State University,

Identity XXII (Fibonacci Q u a r t e r l y ,

State College,

Pa.

Vol. 1, No. 2, A p r i l 1963,

p . 68) s t a t e s :
F n F m - F n - k. F m+k
,. = ( - l ) n ~ k F,k F m +,.k - n
The p r o p o s e d identity is i m m e d i a t e on taking m = n and k = 2.
More g e n e r a l l y , we have
FZ - F _ F
n
n - k n+k

= (-l)n"kF,2
k

for

0 < k < n

Also solved by Marjorie Bicknell, Herta T. Freitag, John E. Homer, Jr., J.A.H. Hunter,
Douglas Lind, Gary C. MacDonald, Robert McGee, C.B.A. Peck, Howard Walton, John
Wessner, Charles Ziegenfus, and the proposer.

1965

E L E M E N T A R Y P R O B L E M S AND SOLUTIONS

157

SUMMING M U L T I P L E S O F SQUARES
B-53

Proposed by Verner E. Hoggatt, Jr., San Jose State College, San Jose,

California

Show t h a t
(2n - 1 ) F ? + ( 2 n - Z)Y* + . . . 4- F ^
_ = F!T
1
2
Zn-1
2n
Solution by James D. Mooney, University of Notre Dame, Notre Dame, Indiana
R e m e m b e r i n g that

I
F, 2 = F F , ,
^
k
n n+1
k=0
we m a y p r o c e e d by i n d u c t i o n .

+ [ > ( n - l ) - 2 ] T\

(2n-3)F2

2
2
n = 1, F , = 1 = F - .

Clearlyfor

[2(n-l) - l] ? \

(2n-4)F2

+ . . . + F ^ ^ ^

. . . + F2n_3 = F

Assume

Then
(2n-l)Fj + . . . + F 2 n _ x = [(2n-3)Fj + . . . + F 2 n _ 3 ]
2n-2
2

2 (v F ? + . . . + F
. = F
9) + F
? +
1
2n-2
2n-l
2n-2

2n-l
F

k=0
F

2n-2
F

2n-2F2n-l

2n-l(F2n-2

<F2n-2

2n-l)2

+ F

2n-1>

2n

2n-lF2n =
=

"

2n-2

'

+ 2F

"

2n-2

F?>
k

k=0
2n-2F2n-l

2n-2F2n-l

+ F

2n-1 =

'

Also solved by Marforie Bicknell, J.L. Brown, Jr., Douglas Lind, John E. Homer, Jr.,
Robert McGee, C.B.A. Peck, Howard Walton, David Zeitlin, Charles Ziegenfus, and
the proposer.

158

April

ELEMENTARY PROBLEMS AND SOLUTIONS


RECURRENCE RELATION FOR DETERMINANTS

B-54

Proposed by C.A. Church, Jr., Duke University, Durham, N. Carolina

Show that the n - t h

order determinant

ax

-1

a2

-1

a.
~3
-1

(n)

n-1
-1
satisfies the r e c u r r e n c e

f(n) - a f(n-l) + f(n-Z)

Solution by John E. Homer, Jr., La Crosse.

for

n > 2.

Wisconsin

Expanding by e l e m e n t s of the n-th column yields the d e s i r e d r e lation i m m e d i a t e l y .


Also solved by Marjorie Bicknell, Douglas Lind, Robert McGee, C.B.A.
Charles Ziegenfus, and the proposer.

Peck,

AN EQUATION FOR THE GOLDEN M E A N


B-55

From a proposal by Charles R. Wall, Texas Christian University,

Show that x
where

- xF

a = (1 + >/fT)/2, F

Solution by G.L. Alexanderson,

For

Ft. Worth, Texas

- F

, = 0 has no solution tog r e a t e r than a,


n-1
is the n-th Fibonacci n u m b e r , and n > 1.

University of Santa Clara,

n > 1 let p(x, n) = x

- xF

- F

California

, , g(x) = x

- x - 1, and

i ,
n-2
n-3 , - n-4 .
, n-k-1 .
. _
, _
x
h(x, n) = x
+x
+2x
+...+F,x
+...+F
..
0x + F
k
n-2
n-1
It is e a s i l y s e e n that p(x, n) = g(x)h(x, n), g(x) < 0 for - l / a < x < a,
g(a) = 0, g(x) > 0 for

x > a, and h(x, n) > 0 for

is the unique positive root of p(x, n) = 0.

x > 0. Hence x = a

Also solved by J.L. Broum, Jr., Douglas Lind, C.B.A. Peck, and the proposer.

1965

ELEMENTARY PROBLEMS AND SOLUTIONS

159

GOLDEN MEAN AS A LIMIT


B-56

Proposed by Charles R. Wall, Texas Christian University,

Let

x, , x, . 0 .
1
Z

be the n - t h Fibonacci n u m b e r .
n
byJ x. .. = f(x, ) w h e r e
k+1
k
f(x) =

For

Ft. Worth, Texas

Let x^ > 0 and define


0

. + xF
.
n-1
n
n > 1, prove that the limit of x, as k goes to infinity e x i s t s and

find the limit.

(See B-43 and B - 5 5 . )

Solution by G.L. Alexanderson,

For

\/F

University of Santa Clara, Santa Clara,

California

n > 1 let p(x)


= Xn - xF
rv

in the proof of B - 5 5 ,
p(x) < 0 for

0 < x < ae

- F . . Let a = (1 + \[5)/2.
As
n
n-1
one s e e s that p(x) > 0 for x > a and that
If x, > a, we then have

(x ) > x F + F
= (x
)
k
k n
n-1
k+1
and so x, > x. , , . It is a l s o c l e a r that x, > a i m p l i e s
k
k+1
k
(x. ^ ) n = x, F + F , > a F + F . = a11
k+1
k n
n-1
n
n-1
and hence x, , , > a. Thus x > a i m pr l i e s x > x, > x^ > . . . > a.
k+1
o
o
1
Z
S i m i l a r l y , 0 < x < a i m p l i e s 0 < x < x. < x_ < . . . < a. In both
'

c a s e s the sequence
x , x, , . . . is monotonic and bounded. Hence x,
n
o
1
k
h a s a limit L > 0 as k goes to infinity. Since L s a t i s f i e s

L = n^Y+rr ,
n-1
n
L m u s t be the unique positive solution of p(x) = 0.
Also solved by Douglas Lind and the proposer.

A FIBONACCI-LUCAS INEQUALITY
B-57

Proposed by G.L. Alexanderson,

University of Santa Clara, Santa Clara,

Let F
and L be the n - t h
n
n
b e r r e s p e c t i v e l y . P r o v e that

Fibonacci and n - t h

(F4n/n)n> L2L6L10...
for all i n t e g e r s

n > 2.

L4n_2

California

Lucas n u m -

160

ELEMENTARY PROBLEMS AND SOLUTIONS

Solution by David Zeitlin,

Minneapolis,

April

Minnesota

Using m a t h e m a t i c a l induction, one m a y show that


n
F

4n

4k-2'

n = 1, 2f

...

k=l
If we apply the well-known a r i t h m e t i c - g e o m e t r i c inequality to the unequal positive n u m b e r s

L OJ
2

L, , L. AJ . . . ,
o
10

L . ,
4n-Z

we obtain for

n = 2, 3,
n
2

4n
__

k=l

4k-2

N/L2L6L10 ...

L ^ ^

which is the d e s i r e d inequality.


Also solved by Douglas hind and the proposer.

XXXXXXXXXXXXXXX

ACKNOWLEDGMENT
It is a p l e a s u r e to acknowledge the a s s i s t a n c e furnished by Prof. V e r n e r
E. Hoggatt, J r . concerning the e s s e n t i a l idea of "Maximal S e t s " and
the line of proof suggested in the l a t t e r p a r t of m y a r t i c l e "On the
R e p r e s e n t a t i o n s of I n t e g e r s as Distinct Sums of Fibonacci N u m b e r s . "
The a r t i c l e

appeared

In Feb*,1965s

H. H. F e r n s

CORRECTION
Volume 3 , Number 1
Page 26, line 10 from bottom of page
V ~+V- / ) +V 7 _ = F Q - F _ = F A = 8
7 , 3 7 , 4 7 , 5
8 7
6
Page 27, lines 4 and 5
F ^ + F . + F , * . . . +F = F ,, -1 (n even)
2 4 6
n
n+1
F Q + F C + F ^ + . . . + F = F ,. -1 (n odd)
3 5 7
n
n+1
ACKNOWLEDGMENT
Both the p a p e r s "Fibonacci R e s i d u e s " and "On a G e n e r a l Fibonacci
Identity, " by John H. Halton, w e r e supported in p a r t by NSF g r a n t
GP2163.
CORRECTION
Volume 3 , Number 1
P a g e 40, Equation (81), the R. H. S. should have an additional t e r m
-

V 2 F

V+2

BASIC PROPERTIES OF A CERTAIN GENERALIZED SEQUENCE


OF NUMBERS
A. F. HORADAM
The University of North Carolina, Chapel H i l l , N. C.

1.

INTRODUCTION

Let a, (3 be the roots of


x 2 - px + q = 0

(1.1)
where

p, q are a r b i t r a r y integers.

Usually,

we think

of a, p as

being real, though this need not be so.


Write
d = (p

( 1 . 2)

- 4q) '

Then
a = (p + d ) / 2 , P= (p - d)/2

(1. 3)
s o that
(1.

a + P = p, aP = q, a - P = d.

4)

Recently [6] , a certain generalized sequence | w j was defined:


(1.5)

j w l = jw n (a, b;p, q)| : wQ = a, w ^ b , w n = p w n 4 - q w n _ 2 ( n

in which
(1.6)

= Acin + Bp n ,

where
(I 1)
11 7

A -

" aP

" a- p

R-

ii5

a a

"

" a -P

whence
(1.8)

A + B = a, A - B = (2b - pa)d - 1 , A B = e d" 2

in which we have written


2
(1.9)

e = pab - qa
161

2
- b .

^ Z)

162

BASIC P R O P E R T I E S OF A CERTAIN
Sequences like j w

example,

Oct.

[ have been p r e v i o u s l y introduced by,

for

B e s s e l - H a g e n [ l ] and T a g i u r i [l l] , though in the available

l i t e r a t u r e I cannot find evidence of much p r o g r e s s from the definition


[11] to have d i s c o v e r e d a few of the r e s u l t s listed h e r e u n d e r .
The p u r p o s e of [6 J was to d e t e r m i n e a r e c u r r e n c e r e l a t i o n for
the k

p o w e r s of w

(k an i n t e g e r ) , that i s , to obtain an explicit

form for
w k (x)

w_k x n
2
n=0

H e r e , we p r o p o s e to e x a m i n e some of the fundamental a r i t h m e t i c a l


p r o p e r t i e s of jw 1 .

No a t t e m p t at all is m a d e to analyze congruence

or p r i m e n u m b e r f e a t u r e s of 5w t . In selecting p r o p e r t i e s to g e n e r alize we have been guided by those p r o p e r t i e s of the r e l a t e d s e q u e n c e s


(see 2. below) which in the l i t e r a t u r e and from e x p e r i e n c e s e e m m o s t
b a s i c . Naturally, the list could be extended as far a s the r e a d e r ' s enthusiasm persists.
It is intended that this p a p e r should be the f i r s t of a s e r i e s investigating a s p e c t s of jw I .

Organization of the m a t e r i a l is a s fol-

lows: in 2 , v a r i o u s s p e c i a l (known) s e q u e n c e s r e l a t e d to Iw [ a r e
introduced,

while in 3. s o m e l i n e a r f o r m u l a s involving

jw I a r e e s -

tablished, and in 4. some n o n - l i n e a r e x p r e s s i o n s a r e obtained. F i n a l 2


iy, in 5 . , some c o m m e n t s on the d e g e n e r a t e c a s e p = 4q a r e offered,,
2.

R E L A T E D SEQUENCES

P a r t i c u l a r c a s e s of 5w I a r e the s e q u e n c e s

jfnj,

j l n [ given by:

(2.1)

w n (1,

p; p,

q) = u n (p, q)

(2.2)

w n (2,

p; p,

q) = V n (p, q)

(2.3)

(r, r + s ; 1, -1) = h
n

(r, s)
n

^u I , 5v t , jh I ,

1965

GENERALISED SEQUENCE OF NUMBERS

(2.4)

wn(1;

1; 1, -1)= f n ( = u n ( l , - l )

(2.5)

wn(2,

1; 1, -1)= l n ( = v n ( l , - l ) = h n ( Z , - l ) ) .

163

= hn(l,0))

Historical information about these second order recurrence sequences may be found in Dickson [3] .
Fibonacci sequence,

il

Of course, Jf I is the famous

I is the Lucas sequence, and Ju [ and Jv I

are generalizations of these, while Jh 1 discussed in [4] is a different


generalization of them.

Chief properties of ju I , Jv [ , Jf [ and il

may be found in, for instance, Jarden [7] , Lucas [81 and Tagiuri [10]
and [l l] , those of Jf I especially being featured in Subba Rao [9j and
Vorob'ev [l 2] .
Two rather interesting specializations of (2. 1) and (2. 2) are the
Fermat sequences

Ju

and the Pell sequences

(3, 2)1 =J2


Ju

-ll

(2, -1)[

and

and Jv

(3, 2)1 = j2

From (1.6), (1.7) and (2.1) - (2.5) it follows that


n+1
(2.6)
'

^
d

v n = an + p n

(2. 7)
(2.8)

fln+l

_ (r + s - r P ^ a " - (r + 8 - r a ^ p "

n+1

(2-9)

_ al

n
(2.10)
v
'

-n+1

"h
^5

= a? + p!1
n i l

wherein
/9 n\
(2.11)
that is, a, , (3
(2.12)

ax = -,

+ N /

+ l[

Jv (2, - 1 ) 1 . (See [l] or [8])..

, opx = 1

a r e the roots of
x 2 - x - 1 = 0.

-^5

164

BASIC P R O P E R T I E S OF A CERTAIN

Oct.

Consequently, by (1.4)
(2. 13)

a x + Px = 1, a 1 P 1 = - 1 , c^ - ^

5.

To a s s i s t the r e a d e r , and as a s o u r c e of r e a d y r e f e r e n c e , the full set


of r e s u l t s for the five s p e c i a l i z a t i o n s of Jw { will often be w r i t t e n
down, a s in (Z. 6) - (Z. 10).
Obviously from (1 . 9),
F

e c h a r a c t e r i z e s the v a r i o u s

j U n j >2 j V n( ' j h n | ' j f n [ '


- r s - s , 1, 5 r e s p e c t i v e l y .

\* n\

We d e r i v e

= ^

sequences.
^

"

^'

By ( 1 . 6), ( 1 . 7) and (Z. 6) we have


(Z0 14)
w = au + (b - pa) u , = bu , - q a u 0 ,
v
r
'
n
n v
' n-1
n-1
^
n-Z
with, in p a r t i c u l a r , the known [8] e x p r e s s i o n s
(Z. 15)
v = Zu - pu , = pu , - 2q u - .
x
'
n
n c n-1 r n-1
^ n-Z
(Ultimately,
of c o u r s e , t h e s e yield
1 = Zf - f . + Zf ~. )
7
7
n
n
n-1
n-Z
Putting n = 0 in(Z. 14) r e q u i r e s the e x i s t e n c e of values for negative s u b s c r i p t s ,

as yet not defined.

Allowing u n r e s t r i c t e d values of

n t h e r e f o r e in ( 1 . 6) we obtain
C

w'

= A a " n + B (3~n

(2.16)
= q

Ki^V-l*

after simplification using


(Z. 17)
x
'

u
-n

=q
^

^ ,
n-Z

which follows from (Z. 6).


Combining (Z. 14) and (Z. 16) we have
(au - bu -,)
' n
n-1'
w
= q
r,
-n H
au + (b - pa u T
r
n
n-1
whence it follows from (Z. Z) - (Z. 5) that
/o i o\
(Z.18)
v
'

(2; 19)
'

-n

= qn v
-n
^
n

w
n

1965

GENERALISED SEQUENCE OF NUMBERS

(2.20)

=(-l)n

\r (u

- u

-n
<2-21>

, ) - su
A
2ll
2li?
r u -f s u
,
n
n-1

165

^ n ^ "

In p a r t i c u l a r ,
w , = A a " 1 + (3" 1 = P a "
-1
q

(2.23)

so that
(2. 24)

(2. 25)

= p

(2..26)

- 0

-1

-l

(2.27)

= 0

(2. 28)

= -1

M a n y of t h e s i m p l e s t

Iw \ a r e

e x p r e s s i b l e in t e r m s

B e s i d e s (2.4) we have
(2.29)

(2.30)

wn(

(2.31)

w n ( 1,

( _ 1

'

1;

-1'

1, - 1 ;

_1)

1, - 1 ) =

<"1)n"1

-f n _ 3

1; - 1 , - 1 ) = ( - l ) n _ 1 f n 3

More generally,
(2.32)

w n (a, b;

(y 2 . 3 3 )

1, - 1 ) = a f n _ 2 +

bf^

(a, b; - 1 , - 1 ) = ( - l ) n k f
- bf
A
I n - Z9
n-1)

Notice that
w n (alf

bi; p

qx) = - w n (a2, b 2 ; p2>

(2. 34)<[provided
a

'ar

2 = "bi'

P2

= p

q2)

of jf 1

166

BASIC P R O P E R T I E S OF A CERTAIN
Some sequences a r e c y c l i c .

(2.35)

Oct.

Examples are

w n (a, b; - 1 , 1)

2
for which a, (3 (= a ) a r e the complex cube r o o t s of 1 and
(2.36)

w n (a, b; 1, 1)

2
for which a, (3 (= a ) a r e the complex cube r o o t s of - 1 .
(2. 35) is cyclic of o r d e r 3 (with t e r m s a, b, - a - b) since

Sequence
= p

=1,

while sequence (2.36) is cyclic of o r d e r 6 (with t e r m s a, b, - a + b,


,
u\
3n 0 3 n
,
6n Q 6n , ,
,, . ^ .
-a, - b , a - b) since a
= p
= - 1 , so a
= p
= l ( n odd m this
c a s e ) . (Refer (1.6)).
G e o m e t r i c - t y p e s e q u e n c e s a r i s e when p .= 0 (so that by (1.5)
w n + 1 = - q w n 4 ) and q = 0 (so that w n + 1 = p w j .
3.

LINEAR P R O P E R T I E S

F r o m (1.5) and (1.6) it follows that


(3.1)

w
wn ,
n-1

(3. 2)

CL

P.. wn - k.

w n + 2 - (p

>

(a
/P
(

if -1 < p < 1 ,
if -1 < a < 1 ,

- q) w n + pq w n ^ x = 0 ,

and
( 3 . 3)

- (p 2 - q) w n + 1 + q 2 w n _ x = 0 .

PWn+2

Repeated use
the sum of the f i r s t

of qw,
n

, = -w, ,, + pw

(k = 1, . . . , n) leads to

terms

n-1
(3. 4) q

2 ] w^ = (p -1) (w 2 + w 3 +
j=0

+ wn) - w n + 1

+ p W]L

whence
(3.5)

(p - q - 1)

n-1
^
j=0

= w n + 1 - w x -(p - 1) (w n - w Q )

1965

GENERALISED SEQUENCE OF NUMBERS

167

while the c o r r e s p o n d i n g r e s u l t s for differences a r e


n-1
(3.6)

q ]T

(-1) J w. = (p +1) (-w 2 + w 3

j=0
+(-1)n_1

n)+(-1)Ilwn+l

+pW

n X

and

(P - q + D X
j

(3.7)

(-1)

w.

"

= ( - l ) n + 1 w n + 1 +w 1 -(p+1) j ( - l ) n + 1 w n + wQ [ .
Replace n by 2n in (3. 4), (3. 5) (3. 6) and (3. 7).
(3.8)

<7=w0+w2+...

+w2n_2

P = wx + w 3 + . . . + w 2 n _ 1

Write

and
(3.9)

Adding and s u b t r a c t i n g (3. 4), (3. 6) give


(3.10)

(1 + q ) (J = - p p - ( w 2 n - w Q )

and
(3.11)

(1 + q ) p = p O +q(w 2 n _ 1 - w_ x )

for the sum of the even - (odd -) indexed t e r m s of ^w \ . C l e a r l y by


(1.5) addition of (3. 10) and (3. 11) yields the sum of the f i r s t 2 n t e r m s
(3. 4) as expected.

Solve (3. 10) and (3. 11) so that

j p 2 - (l+q) 2 [ a = ( l + q ) ( w 2 n - w Q ) - pq ( w ^ ^ - w ^ )

(3.12)
and

j p 2 - ( l + q ) 2 [ 9 = p ( w 2 n - wQ) - q ( l + q ) ( w 2 n _ 1 - w ^ )

(3.13)

Using the a l t e r n a t i v e

expression w

= bu

we have
(

n+1

I n

n-1

i o = w~
\/ w n+^
2 un - nq w,1 un-1,
f w . 0 = w 0 u - nq w~ u ,
V n+3
3 n
2 n-1

, - qau

.
~ (2. 14),

1^8

BASIC P R O P E R T I E S OF A CERTAIN

Oct.

whence
C
, = w ru -n q ^w
\ w n+r
<
i
= w u - q
I
n r
^
n and r . Equations

(3.14)

on i n t e r c h a n g i n g

from (1.5), (2. 1) and (2. 1.-4).

,
r - 1, u n-1
w 1 u 1
n-1 r - 1
(3.14) m a y a l s o be

obtained

Of c o u r s e

w , = w . u ,. - q w . . u , . ,
n+r
r - j n+j
^ r - j - 1 n+j-1
(3.15)
n+j

r-j

n+j-1

r-j-1

also.
F u r t h e r , from ( 1 . 6) and (2. 7) it follows that
/o

, r
wn + .r + q w

i \

(3.16)

n-r

= v
w

r
n
t h a t i s , the e x p r e s s i o n on the left is independent of a, b, n.
r and n in (3. 16) and then set r = 0. Accordingly,
(3. 17)

Interchange

+q w
=a v .
n ^ . -n
n
O b s e r v e a l s o from ( 1 . 6) and (2. 6) that
w . - q
-I

(3.18)

"r

r_1

s
u ,
7 . - q w
s -1
n+s
^
n
s
which [lo] is an i n t e g e r provided s divides r .
Two binomial r e s u l t s of i n t e r e s t m a y be noted.

Firstly,

(1.6) it follows that

(3.19)

2n=("q)

w h e r e we have used the fact

j> ( ->

--<

a^ - pa + q = 0, (3^ - pp + q = 0.

Starting from ( 1 . 3) and ( 1 . 6), we r e a d i l y d e r i v e


2nw

= A(p + d ) n + B(p - d ) n

from

GENERALISED SEQUENCE OF NUMBERS

l6<

[n/2]
n

(3.20) 2 w n = a

pn_2jdZJ(J)
2
J

Z
j=o

[n-lj

+ (2b-pa)

Z
J=0

( *
J

J p ^ ^ d

whence follow the known [ l ] e x p r e s s i o n s


2nun=

(3.21)

[n

(J++11)pn-2Jd2J

j=0
2n " 1 v n =

(3.22)

pn-2Jd2J

(J)

j=0
[n 2]

j=0

(3.23)

2 fn=

2 n _ l l

(3.24)

n+1
i
<2j+V5

n /2]
Z
j=0

2j)

Suitable substitutions in the above r e s u l t s lead to the s p e c i a l c a s e s


jVn|'

for K l *

JM'

jfnj

and j l n j ; for e x a m p l e , for j f n j , in (3.4)

<T+P=f

2n+l-1'

and in (3. 14) with r = n,

+ f

n-l=f2n=

<&
k=0

using (3. 19).


If we w r i t e
(3.25)

% _
W

n+1

so that, by (1.5),
(3.26)

-p- q r

,
'

n-1 - p - q r

n-k'

170

BASIC P R O P E R T I E S OF A CERTAIN

enabling us t o e x p r e s s the limit of the r a t i o a s a continued

Oct.
fraction.

S o m e t i m e s , when q = - 1 , it is notationally convenient to w r i t e


a

- e " = s m h f] + cosh n
Q

'O

= -e

- 7] - sinh T) - cosh n
o
'o
b

o
w h e r e (1.2)
-o
-1
(3. 28)
cosh T) = ' , sinh r\ = , tanh t) - p d
*
'
o 2 '
'o 2
o ^ o
Z e r o suffices signify that q = - 1 .
Combining this hyperbolic notation with the r e m a r k s i m m e d i a t e l y
p r e c e d i n g (3.27), and p r o c e e d i n g to the limit ( r e f e r (3.1)), wevsee that
for

p - 1,

q == - 1 , that i s , for \h I (and its s p e c i a l i z a t i o n s Si \, Si (),

.
r

T-

h n i..l

1
a"1

- e

-r\\

cosh 7| - sinh r\

1 +
1 + 1
1
2
( o b s e r v e that by (2.12) = g
is a root of x + x - 1 = 0 so t h a t
ttl
i
g } leading to the continued fraction. )
F u r t h e r m o r e , (3.27) and (3.28), with (1.5), imply
+ ( ~ l ) n B ) sinh n rj + (A - (-1 ) n B ) cosh n ri
o, n
o x '
o
'o x o % '
o'
'o
Hyperbolic e x p r e s s i o n s for the s p e c i a l i z e d s e q u e n c e s a r e then,
(3130) w

=(A

(2.6), (2.7), (2.9), (2.10),


u
(3.31)

=
n

sinh (n + 1) 77
T
cosh
71
cosh (n + 1)
'o

cosh
o

(n odd)
'
,
x
(n even)

from

1965

GENERALISED SEQUENCE OF NUMBERS


v

(3.32)

'

= 2 s i n h n T)

2 c o s h n r)

with c o r r e s p o n d i n g e x p r e s s i o n s for
i s r e p l a c e d by T] .

A hyperbolic

4.

f ,

(n e v e n )
(n

odd)

respectively,

e x p r e s s i o n for

NON- LINEAR

171

i n w h i c h r\

is given in

[5] .

PROPERTIES

E s s e n t i a l l y , the p r o b l e m in obtaining n o n - l i n e a r f o r m u l a s (as in


t h e l i n e a r c a s e ) i s to d e t e c t t h e a p p r o p r i a t e c o e f f i c i e n t s ( f u n c t i o n s of
k
p , q) of w . B a s i c n o n - l i n e a r ( q u a d r a t i c ) r e s u l t s h a v e a l r e a d y b e e n
r e c o r d e d i n [6 J , n a m e l y ;
(4.
1)
x
'
(4.
2)
v
'

aw

, + ( b - rp a ) w
, . - w
w - qw
, w
,
m+n
m+n-1
m
n
^ m-1
n-1

2
2
a w 0 + v( b - rp a ) w~
, = w - qw
, = w ., w , - q w w 0
2n
' 2n-l
n
^ n-1
n+1 n - 1 ^ n
n-2

iA o\
(4. 3)
x
'

2
n-1
., w
, - w = q
e
n
n+1
n-1
n

O b v i o u s l y , f r o m ( 4 . 3) w i t h
(4, 4)

n = 0,

e = q (wl

w ^ x - wQ)

w h i c h m a y be c o m p a r e d w i t h ( 1 . 9), u s i n g ( 1 . 5) a n d ( 2 . 2 3 ) .
A n e x t e n s i o n of ( 4 . 3) i s , by ( 1 . 6) a n d ( 2 . 6),
/ A r\
(4. 5)
v
'

2
n-r
2
, w
- w = e q
u
,
n
n+r
n-r
n
r-1
r = n in (4.5), we have
w

Putting

2
(4. 6)
x
'

2
+ e u

n
Interchange

and

T = a w0
n-1
2n
in (4.5), then suppose

/ A ~7\
2 .
-n
2
( 4 . 7)
w w
= a + e q
u
,
x
'
n -n
^
n-1
(n = 1 r e d u c e s (4. 7) t o (4. 4 ) . )
Specializations

of ( 4 . 1) a r e ,

r =. 0.

We d e d u c e

on m u l t i p l i c a t i o n b y 2 a n d u s e of

( 1 . 2 ) , ( 1 . 4 ) , ( 2 . 6 ) , ( 2 . 7 ) a n d ( 2 . 15), t h e k n o w n [8] r e s u l t s

172

BASIC P R O P E R T I E S OF A CERTAIN

(4. 8)

Oct.

2 u , 1 = u . 1 v + u
, v
m+n-1
m-1 n
n-1 m

and
(4. 9)
'

2 v . . = v v . + d u 1 u 1 .
m+n
m n
m - 1 n-1
Next, by (4.6), we d e r i v e , using (2. 6), (2.7),

(4.10)
x
'
and

(4. 11)

2 v , = v2 + d2 u 2 1
2n
n
n-1

( 1 . 2) and ( 1 . 4),

u- . , = u , v
2n-l
n-1 n

with
i A i o\
(4. 12)
Again,

2 0 n , 2
2 I 0 n
v . = v - 2q = d u , + 2q
.
2n
n
n-1
^
(4.1) with m = 2n gives an e x p r e s s i o n for w~

which we deduce,
for

from

by (4. 10), (2.6), (2.7) and the r e c u r r e n c e r e l a t i o n

v~ ,
3n

/A i o\
(4. 13)

3n-l
2
n
= v - q

n-1
and
/

V-2
3n

(4. 14)
x
'

= v
v

- 3q
n

n
R e s u l t s (4. 10) - (4. 14) occur in Lucas [8] in a slightly adjusted
notation.
Coming now to the sum of the f i r s t
half of (4. 2).
Write

(4.15)

n t e r m s , we use the f i r s t

n-1

^ = Z w2
j=0

Then, it follows that


(4.16)

2
(1-q) r = a ( 7 + ( b - p a ) p - j ^ n - 1

+ (b

"Pa)

2n-l 1

1965

GENERALISED SEQUENCE OF NUMBERS

173

whence r may be found from (3. 12) and (3. 13).


Repeating the first half of (4. 2) leads to
(4.17)

- q 2 w ^ x = b w 2 n + 1 + <b- p a ) q w 2 n - 1

w ^

From (1. 6), (1.8) and (2.6),


(4. 18)

w
n-r

- w
n+r+t

w .. = q
n+t
^

e u

, u ., ,
r-1 r+t-1

whence t = 0 gives (4.5)


Repiac
Replacing
w
for

by u

in (3. 14) and (3. 15) (with -j

substituted

j) yields
x . - u u - q u , u T=U
.u , . - q u
i u .. ,
n+r
n r
n-1 r-1
n-j r+j
^ n-j-1 r+j-1

whence
u u - u
. u , . = q (u , u , - u
. , u . . ,)
n r
n-j r+j
^ n-1 r-1
n-j-1 r+j-1
(4.20)
x
'

{
^

n-j ,
.
- q J(u. u
... - u
,9.)
^
j r-n+j
r-n+2j
n-j+1
- q J
u. -, u
,. H
j - 1 r-n+j-1

by repeated application of (4. 19) and replacement in the first half of


(4. 19) of n by r-n+j
(un = 1).

Note that

and r

by j to obtain an expression for

(4.20) is

the

special

case

,,.

of (4.18) for which

w =u
so that e = -q (n, r, j in (4. 20) replaced by n - r, n + r + t,
n
n
^
respectively and (2.17) used).
In particular, it follows from (4.20) with j = 1 that
(4. 21)
v
'

, u -> - u ^ u , ~ q
n-1 r-2
n-2 r-1
^

Moreover,
quences
(4.22)
v
'

, .
r-n-1

(4.21)
and w = b u
-, - q a u 0
x
'
n
n-1 n
n-2
(w \ and jw' (

give
&

for the

se-

w' w - w w' = q (a* b - a b!)(u n u ~ - u *, u- , )


n x
n r
n r
' n-1 r-2
n-2 r-1
=q
^

(a'b - a b') u
,
' r-n-1

Cubic expressions in w
derive only the sum of the first
then use (1.5) again.

Thus

are generally quite complicated, so we


n cubes,

Cube both sides of(1.5)and

174

BASIC P R O P E R T I E S OF A CERTAIN
3 3
3 3
(4. 23)
w ,, = p w - q w , - 3 pq w * w w ,.
x
'
n+1 r
n n
n-1
^
n-1 n n+1
But, from (4. 3),

Oct.

(4. 24)
'

, w w ,, = w + q
n-1 n n+1
n ^

ew

,
n

so that from (4. 23) and (4. 24) it follows that


O

(4. 25)

Q.

*2

' "2

w n + 1 + (3 pq - p ) w n + q w ^

= - 3 pq 11 e w n

Now a calculation involving (1.6) and the s u m m a t i o n of g e o m e t r i c


s e r i e s leads to
(4.26)

n-1

j=1

.
J
q w

^
= 3
i-pq+q

jwx - q

w0-qn'

o
(wn-q

w ^ x) \

Write
n-1

(4.27)

w3

co = X

j=0

Combining (4. 25), (4. 26) and (4. 27), we find


(4,28)

(l+3pq-p3+q3)a;

^ES_ jw^q^-q*1" V
1-pq+q

+q

3 3
w
n _ r

3
n

q K ^

3
3
(l+3pq-p ) w Q

A p p r o p r i a t e substitution in the above f o r m u l a s of 4. Lead to c o r responding r e s u l t s for the s p e c i a l s e q u e n c e s (2.1) - ( 2 . 5 ) .


s t a n c e , applying (4. 16) and (4. 28) to if
3

I, we have
11

r^_

co = I J'f
+ f + St-l)
"* f , + z\
x
4 < n-1
n
'
n-2
>

F o r in-

i f 2n-rVi!

respectively.
5.

DEGENERATE CASE

Throughout the a n a l y s i s of the n a t u r e of Jw [ , the hypothesis


2
2
that p $ 4 q h a s been a s s u m e d . But suppose now that p = 4C1. The

1965

GENERALISED SEQUENCE OF NUMBERS

simplest

degenerate

case

175

o c c u r s when p = 2, q = 1 (a = p = 1)

for

which e x i s t s the t r i v i a l sequence (n ^ 0)


(5.1)

v n (2, 1) : 2,

2,

2,

2,

2,

...

...

and the sequence of n a t u r a l n u m b e r s (n > 0)


(5.2)

u n ( 2 , 1) : r,

2,

3,

4,

5,

that i s , u = n+1 and v = 2 . F o r negative


n, (2. 19) i m rp l i e s v = v .
to
n
n
~n
n
that i s , e v e r y e l e m e n t of j u (2, 1) is 2, while (2. 17) i m p l i e s
u

= -u

2,

that i s , like e l e m e n t s of Ju

(2, 1)1 a r e the positive and

negative i n t e g e r s in o r d e r .
Generally, in the d e g e n e r a t e c a s e ,
a=p =

(5.3)

The m a i n f e a t u r e s of the d e g e n e r a t e c a s e , as they apply to j u


and

5v | a r e

Riordan.

discussed

in C a r l i t z [2] ,

with acknowledgement

I
to

Brief c o m m e n t s , as they r e l a t e to Sw I , a r e m a d e in [6 J .

In p a s s i n g , we note that C a r l i t z [2] has e s t a b l i s h e d the i n t e r e s t i n g r e lationship between d e g e n e r a t e

K<P.>(
and the E u l e r i a n polynomial A, (x) which

satisfies the

differential

equation
A ,, (x) = (1 + nx) A (x) + x(l - x) ^ - A (x) ,
n+1
n
dx
n
2
w h e r e A (x) = A, (x) = 1, A~(x) = 1+x, A Ax) = 1 + 4x + x .
Finally, it m u s t be e m p h a s i z e d that jh [ and its s p e c i a l i z a t i o n s
jf | and jl | can have no such d e g e n e r a t e c a s e s , b e c a u s e

- 4q

then equals 5 ( + 0).

REFERENCES
1.

Bessel-Hagen,

E. ,

"Repertorium

Leipzig, 1929, p . 1563.

d e r hBheren M a t h e m a t i c , "

176
2.

BASIC P R O P E R T I E S OF A CERTAIN
Carlitz,

L. , "Generating functions for p o w e r s

Oct.
of c e r t a i n s e -

quences of n u m b e r s , " Duke Math. J . , Vol. 29, 1962, pp. 521-537.


3.

Dickson, L, , " H i s t o r y of the t h e o r y of n u m b e r s , " Vol. 1, New


York, 1952, Chapter 17.

4.

Horadam,

A. F , "A g e n e r a l i z e d Fibonacci sequence, " A m e r .

5.

H o r a d a m , A. F . ,

Math. Month., Vol. 68, No. 5, 1961, pp. 455-459.


"Fibonacci number t r i p l e s , " A m e r .

Math.

Monthly, Vol. 68, No. 8, 1961, pp. 7 5 1 - 7 5 3 .


6.

H o r a d a m , A. F . , "Generating functions for p o w e r s of a c e r t a i n


g e n e r a l i z e d sequence of n u m b e r s , " (to a p p e a r in Duke Math. J . ) .

7.

Jarden,

D. ,

"Recurring

sequences,"

Riveon

Lematematika,

J e r u s a l e m , 1958.
8.

L u c a s , E. , " T h e o r i e des n o m b r e s , " P a r i s ,

1961, Chapter 18.

9.

Subba Rao, K. , "Some p r o p e r t i e s of Fibonacci n u m b e r s , " A m e r .


Math. Month., Vol. 60, No. 10, 1953, pp. 680-684.

10.

T a g i u r i , A . , " R e c u r r e n c e s e q u e n c e s of positive i n t e g r a l t e r m s , "


(Italian), P e r i o d , di Mat. , s e r i e 2, Vol. 3, 1901, pp. 1-12.

11.

Tagiuri, A . ,

"Sequences of positive i n t e g e r s , " (Italian),

ibid.

pp. 97-114.
12.

V o r o b ' e v , N N , "The Fibonacci n u m b e r s , " ( t r a n s l a t e d from the


R u s s i a n ) , New York, 1961.

A. F . H o r a d a m
University of New England, A r m i d a l e , A u s t r a l i a .
XXXXXXXXXXXXXX

REQUEST
The Fibonacci Bibliographical R e s e a r c h Center d e s i r e s that any
r e a d e r finding a Fibonacci r e f e r e n c e , send a c a r d giving the r e f e r e n c e
and a brief d e s c r i p t i o n of the c o n t e n t s . P l e a s e forward all such inf o r m a t i o n to:
Fibonacci Bibliographical R e s e a r c h C e n t e r ,
Mathematics Department,
San J o s e State College,
San J o s e , California

NON-FIBONACCI NUMBERS
H. W. GOULD
West V i r g i n i a University, Morgantown, W. V i r g i n i a

In o r d e r to u n d e r s t a n d the p r o p e r t i e s of a set it is often worth


while to study the c o m p l e m e n t of the set.

When The Fibonacci A s s o c i a -

tion and this Q u a r t e r l y w e r e being e s t a b l i s h e d ,

the w r i t e r began to

think about non- Fibonacci n u m b e r s a s well a s about Fibonacci n u m b e r s ,


but what is known about non- Fibonacci n u m b e r s ? With the hope of gene r a t i n g m o r e i n t e r e s t in n o n - F i b o n a c c i n u m b e r s ,

I posed as the f i r s t

p r o b l e m in this Q u a r t e r l y , p r o b l e m H - 1 , the question of finding a form u l a for the n - t h n o n - F i b o n a c c i n u m b e r .

The p u r p o s e of the p r e s e n t

p a p e r is to d i s c u s s the p r o b l e m and give a solution to it.


We begin with the concept of c o m p l e m e n t a r y s e q u e n c e s .
quence is an o r d e r e d seto

A se-

Two s e t s of n a t u r a l n u m b e r s , say A and

B, a r e called c o m p l e m e n t a r y if they a r e disjoint and t h e i r union is the


set of all n a t u r a l n u m b e r s .

Many e x a m p l e s a r e available:

Even n u m -

b e r s and odd n u m b e r s ; p r i m e s and n o n - p r i m e s ; k-th p o w e r s and non


k-th powers.

But the r e a d e r m a y not r e a l i z e that f o r m u l a s can be

w r i t t e n down for such s e q u e n c e s .

Of c o u r s e , even and odd n u m b e r s

a r e g e n e r a t e d e a s i l y by 2n and 2 n - l

w h e r e n is any n a t u r a l n u m b e r ,

but it is not as well known that a bonafide formula for the n - t h

non

k - t h power is given by the e x p r e s s i o n

n+

[Vn+

[^~

k 2

w h e r e s q u a r e b r a c k e t s indicate the i n t e g r a l p a r t of a n u m b e r .

Such a

formula i s quite e n t e r t a i n i n g , and is a s p e c i a l c a s e given by Lambek


a n d M o s e r [l l] in a g e n e r a l study of c o m p l e m e n t a r y s e q u e n c e s .

They

give seven e x a m p l e s , as well as a g e n e r a l r e s u l t .


A r e m a r k a b l e p a i r of c o m p l e m e n t a r y sequences was d i s c o v e r e d
about forty y e a r s ago by Samuel Beatty at the U n i v e r s i t y of Toronto.
He posed his d i s c o v e r y a s a p r o b l e m in the A m e r i c a n M a t h e m a t i c a l
Monthly [2] . We m a y state B e a t t y ' s t h e o r e m in the following equivalent

177

178

NON-FIBONACCI NUMBERS

form,

Oct.

If x and y a r e i r r a t i o n a l n u m b e r s such that

l / x + 1/y = 1,

then the sequences [nx] and [ny] , n = 1, 2, 3, . . , a r e c o m p l e m e n t a r y .


This t h e o r e m has been r e d i s c o v e r e d a n u m b e r of t i m e s since
1926,

The s h o r t list of r e f e r e n c e s at the end of this p a p e r will give

some idea of what is known about c o m p l e m e n t a r y s e q u e n c e s .


r e s u l t has been fairly popular in Canada.

Beatty's

B e s i d e s the w o r k in Canada

by Lambek and M o s e r , t h e r e was the w o r k of Coxeter, and the m a s t e r ' s t h e s i s by Ian Connell (published in p a r t in [3 J ).

The i n t e r e s t i n g

extension by Myer Angel [ i j was w r i t t e n when he was a second y e a r


student at McGill U n i v e r s i t y .

Our m a i n i n t e r e s t h e r e is in the 1954

p a p e r of Lambek and M o s e r .
Let f(n), n= 1, 2, 3, . . . , be a n o n - d e c r e a s i n g sequence of p o s i tive i n t e g e r s and define, as in [l Ij and [8, e d i t o r ' s r e m a r k s ] , the ! inverse1 f

by

f (n) = n u m b e r of k such that f(k) < n =

1 < k
f(k) < n
is the d i s t r i b u t i o n function which one would expect to study

Thus f

in connection with any s e q u e n c e .

If f defines the sequence of p r i m e

n u m b e r s , then f' (n) = ^ ( n - l ) = n u m b e r of p r i m e s < n.


f

= f.

1 .

We shall a l s o define

F(n) = f ' ( n + l ) .

Note a l s o that

Next, define r e c u r s i v e l y

F Q (n) = n; F ^ n ) = n + F ( F k ^ ( n ) ) ,

k > 0 .

M o s e r and Lambek showred that if Cf(n) is the sequence c o m p l e m e n t a r y


to f(n), then
Cf(n) = k L

F k (n)

What is m o r e , they showed that the sequence

F, (n) a t t a i n s its limit

Cf(n) in a finite n u m b e r of steps when this limit is finite.

In fact one

need not go beyond k = Cf(n) - n.


Thus the n - t h n o n - p r i m e n u m b e r is the limit of the sequence
n + fl'(n), n+ n(n + n"(n)), . . . .
the limit.

Often two steps a r e sufficient to

n,

attain

Thus the n - t h n a t u r a l n u m b e r which is not a p e r f e c t k - t h

power is given by the e x p r e s s i o n enunciated at the outset of this p a p e r .

1965

NON-FIBONACCI NUMBERS

The n - t h n a t u r a l n u m b e r not of the form [e

179

| with m > 1 is

n + [log(n + 1 +[log(n+l)] ) ] .
As for the Fibonacci and n o n - F i b o n a c c i n u m b e r s , let f(n) = f
'
n
be a Fibonacci n u m b e r , defined r e c u r s i v e l y by f ,. = f + f , with
1 J
n+1
n
n-1
f. = 1 , f? = 2, Let g d e s i g n a t e the n o n - F i b o n a c c i n u m b e r s .
The
following
will i l l uF(n)
s t r a t e the
involved.
n
f tableHn)
A calculations
B
D
C
n
~ -f

_
_
__
2
2
1
3
3
1 [
2
1
2 1
2
4
3
5
4
3
2
3
3
6
4
7
4
5
4
3
3
7
4
8
5
8
3
4
5 :
5
10
9
5
13
4
6
4
10
5
11
5
21
4
4
11
7
5
12
5

nsE [

j 0, 67

2. 10

1 2.95

3.55

10

| 4.02

11

4.39

12

4.71

34

13

14

14

4.99

9
10

55

14

15

15

5. 24

15

16

16

5.45

11
12

89
144

16

17

17

5. 65

233

17

18

18

5.84

13

377

19
20

6.00

19
20

610

19
20

14

5
6

15
16

987

21

22

22

6. 30

22

23

23

23

24

24

24

25

19
20

25

26

25
26

i 6.67

j
i

26

27

27

j 6.90

21

i
I

28

28

28

7.00

7
7

29
30

7
7

29
30

7.09

29
30

j 7. 19

24

31

31

31

j 7. 28

25
26

1
i

32

32

1 7-36

32
33

33

33

j 7.44

27

34

35

35

! 7.52

17
18

22

23

6. 15

6.43
6.55

j 6. 79

180

NON-FIBONACCI NUMBERS

Q ct.

In the table, successive columns indicate the steps in evaluation


of the limit

= Cf(n) as follows:
A = n + F(n),
B = F(n + F(n)),
C = n + F(n + F(n)),
D = F(n + F(n + F(n))),
E = n + F(n + F(n + F(n)))

Three iterations were found necessary to generate the non-Fibonacci


numbers

g , at least up to n = 40.

It is left as a research problem

for the reader to determine if more than three iterations are ever
necessary.
It is evident that to obtain an elegant formula for
then two problems:

we have

(a) the number of steps required to find

Cf(n); (b)

a neat formula for the distribution function


inverse

F(n) or equivalently the

f (n).

The study of F or f

corresponds to the study of the distribu-

tion of prime numbers, but because of the regular pattern of distribution we can supply a fairly neat.formula for

F(n).

It was noted by K,

Subba Rao [13] that we have the asymptotic result:


F(n)
~ \^MJL 9
x
'
log a

a s

n-^oo

where
1 + \/5
a = - .
As a matter of fact one can prove much more.
THEOREM.

We have the following

Let F(n) = number of Fibonacci numbers

^ n.

Then

F(n) ~ !"Qg n + log& J5 - 1 = 2 . 08 l o&g n + 0 . 67


'
log a
a
and, for

n > n , F(n) is the greatest integer 1 this value0

in the table gives the value of the expression


computed from a standard 10-inch slide rule.

Column F

2.08 log n + 0.67

as

Even this crude cal-

culation is good enough to show how closely the formula comes to F(n).

1965

NON-FIBONACCI NUMBERS

181

Thus we have the following a p p r o x i m a t e formula for the n - t h


non-Fibonacci number:
g

= n + F(n + F(n + F(n) ,

with
F(n) = [log a n + \

log a 5 - l ]

for

n > 2 ,

= [ 2 , 0 8 log n 4- 0, 67]
We shall conclude by noting some c u r i o u s g e n e r a t i n g functions for
the d i s t r i b u t i o n function (or i n v e r s e )
sequence

of positive i n t e g e r s
x

2
n=l

xf

(n)

f (n).

F o r any n o n - d e c r e a s i n g

f(n), we have [ 8 , e d i t o r ' s

= (1 - x)

remarks]

2
f(n) x n ,
n=l

and

f(k)

k=l

j=l

J,K

f(n)

j=l

k=l

+ (j)

J,K

the l a t t e r identity holding for an a r b i t r a r y a r r a y of n u m b e r s

A. , ,

being m e r e l y an example of s u m m i n g in the one c a s e by rows and in


the other c a s e by columns f i r s t .

As an example with application to

f o r m u l a s involving the F i b o n a c c i n u m b e r s we m a y note that


f

J,K
J K
k=l j=l
j=l k=l + F ( j - l )
'
In this formula, take A
= 1 identically. Then we find the formula

J> k

f
2
k=l

F ( k - 1) = n f n -

fn+2 + 2 ,

(F(0) = 0)

this being but one of m a n y i n t e r e s t i n g r e l a t i o n s connecting f

and

F(n).

and

F r o m T h e o r e m 2 of [l l] we have that the s e q u e n c e s

n + F(n-l) are complementary.

n +f

The r e a d e r m a y find it of i n t e r e s t to

develop the c o r r e s p o n d i n g f o r m u l a s for n o n - L u c a s n u m b e r s or other

182

NON-FIBONACCI NUMBERS

r e c u r r e n t sequences,

Oct.

In a forthcoming p a p e r [lO] Holladay has given

a v e r y g e n e r a l and c l o s e l y r e a s o n e d account of some r e m a r k a b l e r e sults for c o m p l e m e n t a r y s e q u e n c e s .

If a p e r s o n a l r e m a r k be allowed,

his p a p e r is an outgrowth of d i s c u s s i o n s concerning p r o b l e m H - l and


the application of c o m p l e m e n t a r y s e q u e n c e s to c e r t a i n p r o b l e m s in
game t h e o r y .
As a final r e m a r k ,

t h e r e is the question of the d i s t r i b u t i o n of

n o n - F i b o n a c c i n u m b e r s and identities which they m a y satisfy.

It is

hoped to d i s c u s s other p r o p e r t i e s of n o n - F i b o n a c c i n u m b e r s and other


f o r m u l a s for them in a l a t e r p a p e r .

REFERENCES
1.

Myer Angel, P a r t i t i o n s of the n a t u r a l n u m b e r s , Canadian Math.


B u l l . , 7(1964), 219-236.

2.

Samuel Beatty,

P r o b l e m 3177(=3173), A m e r .

Math.

Monthly,

3.

Ian G. Connell, Some p r o p e r t i e s of Beatty s e q u e n c e s ,

Canadian

4.

H. S. Mo Coxeter,

33(1926), 159; solution, 34(1927), 159-160.


Math. B u l l . , 2(1959), 181-197.
The golden section, phyllotaxis, and Wy-

thoff's g a m e , Scripta M a t h e m a t i c a , 19(1953), 135-143.


5.

Ky Fan,

P r o b l e m 4399, A m e r . Math. Monthly,

57(1950), 343;

solution, 59(1952), 4 8 - 4 9 .
6.

E. No Gilbert,

Functions which r e p r e s e n t all i n t e g e r s , A m e r .

Math. Monthly, 70(1963), 736-738; Acknowledgement, 1082.


7.

H. W. Gould, P r o b l e m H - l , Fibonacci Q u a r t e r l y , 1(1963), No. 1,


p . 46.

8.

H W. Gould,
L. M o s e r ,

G e n e r a l i z a t i o n of a b r a c k e t function formula of

Canadian Math.

Bull.,

6(1963), 275-277; E d i t o r ' s

c o m m e n t , 277-278.
9.

H. D. G r o s s m a n ,

A set containing a l l i n t e g e r s , A m e r . Math.

Monthly, 69(1962), 5 3 2 - 5 3 3 .
10.

J o h n C . Holladay, Some convergent r e c u r s i v e s e q u e n c e s , h o m e o m o r p h i c i d e n t i t i e s , and inductively defined c o m p l e m e n t a r y s e q u e n c e s , To be published, 40-page m a n u s c r i p t .

1965
11.

NON-FIBONACCI NUMBERS

J e L a m b e k a n d L . M o s e r , I n v e r s e and c o m p l e m e n t a r y sequences
of n a t u r a l n u m b e r s , A m e r , Math. Monthly,

12.

183
61(1954),

454-458,

R. Sprague, Ein Satz u b e r Teilfolgen d e r Reihe d e r n a t u r l i c h e n


Zahlen, Math. Annalen, 115(1938), 153-156.

13.

K. Subba Rao, Some p r o p e r t i e s of Fibonacci n u m b e r s , BulL , Calcutta Math. S o c , 46(1954), 253-257.

xxxxxxxxxxxxxxx
C o r r e c t i o n s to "Summation F o r m u l a e for Multinomial Coefficients"
by Selmo Tauber, Vol. Ill, No. 2:
(5) line 3 (p. 97)

N+l

(6) last line (p. 97)

I ^ X ^ , . . . etc
a=l

(8) lines 3 and 4, upper index of m u l t , coeff.

(p. 99)

N+h+1
N+q-1

RENEW YOUR SUBSCRIPTION! ! .'


All s u b s c r i p t i o n c o r r e s p o n d e n c e should be a d d r e s s e d to B r o t h e r U,
Alfred, St. M a r y ' s College, Calif. All checks ($4.00 p e r year) should
be m a d e out to the Fibonacci A s s o c i a t i o n or the Fibonacci Q u a r t e r l y ,
M a n u s c r i p t s intended for publication in the Q u a r t e r l y should be sent
to V e r n e r E. Hoggatt, J r . , M a t h e m a t i c s D e p a r t m e n t , San J o s e State
College, Sari J o s e , Calif. All m a n u s c r i p t s should be typed, doublespaced. Drawings should be m a d e the s a m e size as they will a p p e a r
in the Q u a r t e r l y , and should be done in India ink on e i t h e r vellum or
bond p a p e r . Authors should keep a copy of the m a n u s c r i p t sent to the
editors.
The Q u a r t e r l y is e n t e r e d as t h i r d c l a s s m a i l at the St. M a r y ' s College
P o s t Office, California, as an official publication of the " Fibonacci
Association.

TWO FIBONACCI CONJECTURES


DMITRI THORO
San Jose State College, San Jose, California

Consider the p r o b l e m of solving, in positive i n t e g e r s , the following Diophantine equation (suggested by the Editor):
(1)

+ F n + 1 y = x2 + y2.

Fnx

F i r s t let us note that (1) always h a s the t r i v i a l solution (F , F , , ) ,


3
n
n+1'
i. e. , x = F , y = F ,, . Does (1) e v e r have a n o n - t r i v i a l solution? If
n is fixed,

we know from analytic g e o m e t r y that t h e r e a r e at m o s t a

finite n u m b e r of solutions.

However, we shall soon see that for i n -

finitely m a n y n (1) h a s at l e a s t two n o n - t r i v i a l solutions.


T h e o r e m 1.
If n > 1 and n = 1 (mod 3), then
F
n+2,
2

F
n+2
2

and
F

F
n+2,
2

n-1
2

a r e n o n - t r i v i a l solutions of (1).
Proof.

Since n = l

(mod 3), F

=F

+2

;=0 (mod 2) which g u a r a n t e e s

that the quotients involved a r e indeed i n t e g e r s .

One m a y i m m e d i a t e l y

verify that they satisfy (1).


T h e o r e m 2.

If

(x , y ) i s a solution of (1), than u = 2x


o
o
V = 2yJ - F ,, is a solution of
o
n+1
u 2 + v 2 = F

(2)
Proof.

If
,1)

This is an i m m e d i a t e consequence of the identity ( L u c a s , 1876)


F

2n

- F ,
o n

+ F * = F 9 ,. .
n+1
2n+l

u = u o and v = v o is a solution of (2) with (u o, v o') 4' {\,


P
(or any of the other 7 solutions of (2) obtained bychanging signs

or i n t e r c h a n g i n g F

and F

) w e shall call (u ,v ) a n o n - t r i v i a l solu-

tion of (2).
184

1965

TWO FIBONACCI CONJECTURES

T h e o r e m 3.

If

n ^ 1

185

(mod 3), then (1) h a s a n o n - t r i v i a l solution if

and only if (2) h a s a n o n - t r i v i a l solution.


Proof.
(a)
u = 2x

If (x , y ) is a n o n - t r i v i a l solution of (1), then by T h e o r e m 2,

o
then x
o
If 2x o
since F
(b)

- F , v = 2yJ * F (1 is a solution of (2). If 2x - F = F


n
o
n+1
o n
n,
= F (and hence Jy = F ,, or 0) or x = 0, a c o n t r a d i c t i o n .
n
o
n+1
'
o
F = F . , , x = F , T/ / 2 or x
<0 which is i m pc o s s i b l e
n
n+1
o
n+2
o
,~ is odd and we a r e c o n s i d e r i n g only positive solutions of (1).

Let us a s s u m e

>0,v
>0 is a n o n - t r i v i a l solution of (2).
v
o
o
'
F
is even if and only if w = 0 (mod 3); thus by hypothesis F^ ,, is
w
2
2
2n+l
odd. But F T ., = u + v , hence u
and v
m u s t be of different
2n+l
o
o
o
o
p a r i t y . M o r e o v e r , for the s a m e r e a s o n F
and F .. m u s t a l s o be
of different p a r i t y .

Thus (interchanging n a m e s if n e c e s s a r y ) we m a y

be s u r e that

+F

n,

2
is an i n t e g r a l solution of (1).
u

+ F

,,
n+1
2
~

If
o

+ F

= F

we would, a s before, get a c o n t r a d i c t i o n .


The r e a d e r is invited to show that the n u m b e r of n o n - t r i v i a l solutions of (1) is always even.
Now the p r o b l e m of r e p r e s e n t i n g a n u m b e r a s the sum of two
s q u a r e s h a s r e c e i v e d c o n s i d e r a b l e attention.

The following

result,

known to F e r m a t and o t h e r s was proved by E u l e r :


2
If N = be
> 0 , w h e r e b is s q u a r e - f r e e , then N i s r e p r e s e n t a b l e
a s the sum of two s q u a r e s if and only if b h a s no p r i m e f a c t o r s of the
form

4k + 3.
T h e o r e m s on the n u m b e r of such r e p r e s e n t a t i o n s can be found in

v i r t u a l l y e v e r y introductory, text on n u m b e r t h e o r y .
Thus by T h e o r e m 3 if n | 1
form

(mod 3)

and F ^ ,- is a p r i m e of:

4k+l, the only solution of (1) in positive i n t e g e r s is

since e v e r y p r i m e of the form


s e n t i a l l y only one way.

4k+l

(F , F

.)

is the sum of two s q u a r e s in e s -

186

TWO FIBONACCI CONJECTURES

Oct.

It is i n t e r e s t i n g to note that the p e r t i n e n t identity


(a 2 + b 2 ) ( c 2 + d 2 ) = (ac bd) 2 + (ad + b e ) 2
was given by F i b o n a c c i in his Liber Abaci of 1202.
to expedite n u m e r i c a l i n v e s t i g a t i o n s .

However,

This can be u s e d

one needs to b e w a r e

(or at l e a s t be a w a r e ) of such a c c i d e n t s a s the following:


Let

n >o, n = 2 (mod 3);

(i)

If n < 32 (and n f 17), then 2n+l

(ii)

If n < 17, both 2n+l

and

F?

,,

is a p r i m e .
are primes!

Another useful r e s u l t is
T h e o r e m 4.

F~

, E 1, 2, or 5 (mod 8).

2
2
Proof.
We shall use the identity F ? ,, = F + F , . If g is odd,.
then g 2 = 1 (mod 8). Thus if F and F ,, a r e both odd, F_
=2
& *
'
n
n+1
2n+l
(mod 8). Since two consecutive Fibonacci n u m b e r s a r e r e l a t i v e l y
prime,

the only r e m a i n i n g possibility is that

and

a r e of

different p a r i t y ; in this c a s e we get F ? , = 1 or 5 (mod 8).


The r e a d e r m a y prove that in g e n e r a l F ^ 4 (mod 8).
Finally, this p r o b l e m s u g g e s t s the following conjectures:
Conjecture 1.

T h e r e a r e infinitely m a n y values of

for which (1)

h a s only a t r i v i a l solution.
Conjecture 2.
F ? + 1 is n e v e r divisible by a p r i m e of the form 4k+3.
xxxxxxxxxxxxxxx
jNOTICE TO A L L SUBSCRIBERS! ! !
p l e a s e notify the Managing Editor AT ONCE of any a d d r e s s change.!
The P o s t Office D e p a r t m e n t , r a t h e r than forwarding m a g a z i n e s mailed]
t h i r d c l a s s , sends them d i r e c t l y to the d e a d - l e t t e r office. Unless the
a d d r e s s e e specifically r e q u e s t s the Fibonacci Q u a r t e r l y be forwarded!
[at f i r s t c l a s s r a t e s to the new a d d r e s s , he will not r e c e i v e it. (This|
[will u s u a l l y cost about 30 cents for f i r s t - c l a s s p o s t a g e . ) If possible,!
b l e a s e notify us AT LEAST THREE WEEKS PRIOR to publicationj
d a t e s : F e b r u a r y 15, April 15, October 15, and D e c e m b e r 15.

RENEW YOUR SUBSCRIPTION! ! !

FIBONACCIOUS FACTORS
ROBERT B. ELY, III

1.

INTRODUCTION

In e a r l i e r i s s u e s of the Q u a r t e r l y t h e r e have been shown and


p r o v e n a n s w e r s to the following questions about the basic s e r i e s (1, 1,
2, 3, 5 - - - ) .

(1)

By what p r i m e s a r e the v a r i o u s t e r m s , U ., d i v i s i b l e ?

(2)

At what points do v a r i o u s p r i m e s f i r s t a p p e a r a s f a c t o r s ?

(3)

At what p e r i o d s do they r e a p p e a r ?
In this p a p e r we deal with a n s w e r s to the s a m e questions as to

the g e n e r a l s e r i e s (a, b, a + b, a + 2b, 2a + 3b


20

).

PERIODS OF R E A P P E A R A N C E ARE THE SAME

Our t a s k is simplified if we a n s w e r the last question first:


If k is the p e r i o d at which a p r i m e r e p e a t s its z e r o r e s i d u e s
in the b a s i c s e r i e s , k is a l s o the period of z e r o r e s i d u e s in any
general series.
Suppose that a p r i m e f i r s t divides the nth t e r m of a given s e r i e s (a, b, a + b

) and let the

(which we h e r e a f t e r a b b r e v i a t e to
borhood a s

( n - l ) t h t e r m be c.
M

[p") the s e r i e s r u n s in this

c, 0, c, c, 2c, 3c e t c .

means

neigh-

The t e r m s after the z e r o a r e

those of the b a s i c s e r i e s each multiplied by c.


also c x i o

Then modulo p,

Now if x ^ i o

if c = o [p. Again, if x = o [p, so a l s o cx= o [p.

that in the two s e r i e s (1, 1, 2, - - - ) and (c, c, 2c

[p, so
This
) the

z e r o s a p p e a r at the s a m e t e r m s
3.
(1)

SUMMARY OF PREVIOUS RESULTS AS TO FIRST APPEARANCES


T h e r e a r e some t e r m s of the basic s e r i e s divisible by any p r i m e

one m a y choose,
(2)
v

The t e r m

U , . . . is divisible byJ U , U,
abc
a
b

U 1 2 * 144
is divisible by
187

U ...
c

E. G.

188
U

Oct.

FIBONACCIOUS FACTORS

u2= i
U3=2
U4= 3
U6=8

(3)

Such a t e r m

U .,

for which n is c o m p o s i t e ,

may also

have

other f a c t o r s , called " p r i m i t i v e p r i m e d i v i s o r s ; " and the g e n e r a l form


of t h e s e p r i m e s is d e t e r m i n e d by the following r u l e s (but t h e i r identity
m u s t be found by t r i a l and e r r o r ) .
(A) If n is odd; p is of the form

2 kn 1

(B) If n = 2 ( 2 r + l ) ; p is of the form n k . l


(C) If n = 2 m (2 r + 1); p is of the form n k / 2 - 1
E x a m p l e s a r e listed in the F e b r u a r y 1963 Q u a r t e r l y at pp. 4 4 - 4 5 .
(4)

The fact that n is p r i m e does not imply that U

is p r i m e .

E.g.,

U U 19 = 4181 = 37 x 113; even though 19 is p r i m e . However, the conv e r s e is t r u e : If U. is p r i m e , so a l s o is n.


n
^
(5)
The even p r i m e , 2, is a factor of e v e r y t h i r d t e r m of the s e r i e s ;
and the odd p r i m e 5 is a factor of e v e r y 5th t e r m .
(6)

All other odd p r i m e s a r e of the f o r m s 1 and 3 [10.

They

a p p e a r and r e a p p e a r as f a c t o r s according to the following r u l e s :


(a) If p= 1 [10, it will f i r s t a p p e a r when the n of U. = jJ
d being

some positive integer; and will r e a p p e a r e v e r y nth

term

thereafter;
D 4" 1

(b) If p5 3

[10, it will f i r s t a p p e a r when n = --3, and e v e r y

nth t e r m t h e r e a f t e r , d again being some positive i n t e g e r .


7
h
3 divides U8
U4 and e v e" r8y t4th
term thereafter
11
10th "
uio
"
M
13
"
7th
"
U?
17
U9
"
9th
"
19
18th "
U18 "
(c) The r u l e s for d e t e r m i n i n g the d i v i s o r ,
have not yet been given.

of p =fc 1 in (6)

Examination of the p r i m e s l e s s than 80 give

d = 1, 2 or 4 in all c a s e s except 47, w h e r e it is 3.


range from

d,

E. g. ,

p = 2,000 to 3,000,

However,

in the

given in the F e b r u a r y 1963 i s s u e at

pp. 36-40, d h a s values ranging from 1 to 78.

1965
(7)

FIBONACCIOUS FACTORS

Nothing has thus far been said about the a p p e a r a n c e s and p e r i o d s

of c o m p o s i t e f a c t o r s ,
4.
(1)

189

ab (a / b ) , nor f a c t o r s which a r e p o w e r s , p C .

NEW ANSWERS TO THE QUESTION OF FIRST APPEARANCES


"By what factors a r e the t e r m s of the g e n e r a l s e r i e s (a, b, a + b,

a + 2b, + 3b . . . ) d i v i s i b l e ? "
It can be shown that if A, B and C denote any t h r e e s u c c e s s i v e
2
t e r m s in this s e r i e s , then B - AC = a constant, no m a t t e r which
t h r e e t e r m s a r e chosen,

and no m a t t e r what the values of A and

(the f i r s t two t e r m s ) .
Specifically,

work on the f i r s t few t e r m s of the g e n e r a l s e r i e s

shows what this constant m u s t be


b

- a (a + b) = b

- ab - a

or (a + b ) 2 - b (a + 2b) = a 2 + 2ab + b 2

- ab - a b 2

= - b + ab + a
2
2
= -(b - ab - a )

How can we m a k e use of this constancy of B


the p o s s i b i l i t y of a given p r i m e ,
general s e r i e s ?
term,

p,

AC to d e t e r m i n e

as a factor of some t e r m in the

By changing the equation to a congruence [p.

If any

C, of the s e r i e s is divisible by p; then C and its two i m m e d i a t e

p r e d e c e s s o r s m u s t satisfy the congruence


B 2 - ACE (b 2 - ab - a 2 )

[p

But we a r e a s s u m i n g C E O [p. This e l i m i n a t e s the t e r m - AC. Hence


2_
2
2
we m u s t have B - . (b - ab - a ) [p.
In other w o r d s , once we know the f i r s t two t e r m s , a and b of
a g e n e r a l s e r i e s ; we know that the only p o s s i b l e f a c t o r s for t e r m s of
2
2
the s e r i e s a r e those for which (b - ab - a ) is a q u a d r a t i c r e s i d u e .
P r i m e s of which this is not t r u e cannot be the modulus in the congruence
B 2 E (b 2 - ab - a 2 )

[p.-

190

FIBONACCIOUS FACTORS
However,

it does not folLow from the n e c e s s i t y of this condition

that it is a l s o sufficient.
Nevertheless,

Oct.

E. g. , 1, 4, 5 . . . is n e v e r divisible by 89.

B r o t h e r Alfred h a s shown that t h e r e a r e some p r i m e s

which a r e f a c t o r s of a l l F i b o n a c c i o u s s e r i e s .
(2)

We can no longer say that U ,

a s a single example will show.


115, 186, 301.
(3)

is divisible by U , Uv and U ,

Consider 3, 7, 10, 17, 27, 44, 7 1 ,

U 1 Q = 301 is divisible by U 2 = 7, but not by IL = 27.

Neither can we say of a g e n e r a l s e r i e s that if U.

too is n.

Vide 2, 5, 7, 12, 19, 31 . . . for which U/

is p r i m e , so
is p r i m e but 6

is not.
(4)

(a) Nor do we have in the g e n e r a l s e r i e s a set of p r i m i t i v e p r i m e

f a c t o r s , in view of (2) above.


(b) Thus we a r e fairly limited, as to r u l e s for the f o r m s of c e r tain, p o s s i b l e or i m p o s s i b l e p r i m e f a c t o r s of the g e n e r a l s e r i e s .

We

m a k e h e r e only two o b s e r v a t i o n s :
(i) F o r p r i m e s of the form
a r e s i d u e for any value of a.

p = 4k + 3, e i t h e r

a or

-a

Hence t h e s e p r i m e s a r e p o s s i b l e , but not

n e c e s s a r i l y c e r t a i n f a c t o r s of any g e n e r a l s e r i e s .

(ii) On the other hand, for p r i m e s of the form


t h e r e can be values of a for which n e i t h e r
E . g . , neither

2 or

a nor

p = 4k + 1,

- a is a r e s i d u e .

-2 is a r e s i d u e [5; and n e i t h e r 2 nor 5

6 a r e r e s i d u e s [13.

is

Hence t h e s e p r i m e s a r e i m p o s s i b l e f a c t o r s

nor
of

g e n e r a l s e r i e s for which the initial t e r m s a r e c o r r e c t l y chosen.


E . g . , no t e r m s of the s e r i e s 1, 63, 64, 127 a r e e v e r divisible by
5,

11, 13 or 17, since (63 Z - 64) = (3969-64) = 3905 i s a non-

r e s i d u e of e a c h of t h e s e p r i m e s .
Hence let us put a s i d e for the m o m e n t the m o r e p a r t i c u l a r r u l e s
of f o r m s of f a c t o r s of the g e n e r a l s e r i e s , and t u r n to the place of f i r s t
a p p e a r a n c e of p o s s i b l e f a c t o r s .

The i n t e r v a l s of r e a p p e a r a n c e a r e as

in the basic s e r i e s .
(5)

F i r s t let us review 2 and 5.

only four p a t t e r n s , depending


1, 1, 0, 1,
0, 0, 0, 0,
1, 0, 1, 1,
0, 1, 1, 0

If any s e r i e s is reduced

on choice of initial t e r m s
1, 0, 1, 1, 0, . . . . .
0, 0, 0, 0
......
0, 1, 1, 0
.......

2, we have

1965

FIBONACCIOUS FACTORS

191

That is to say: one of the f i r s t t h r e e t e r m s m u s t be even; and t h e r e after e i t h e r all or e v e r y 3rd t e r m is even.
F o r 5, the situation is a Little m o r e complex,,

Actual c o m p u t a -

tion of f i r s t a p p e a r a n c e s for the v a r i o u s combinations of r e m a i n d e r s


of the f i r s t two t e r m s enables us to m a k e the following table:
If the second t e r m h a s a r e m a i n d e r of
0
and the
first a
remainder
of

1 2
1

4
.1

f 2

I 2

1 2

rr"
2

[5

the e n t r i e s show the n u m b e r of the s m a l l e s t t e r m divisible by 5, w h e r e


N signifies

nonee "

Thus we see that 5 m a y f i r s t a p p e a r as a factor

of any t e r m from the 1st to the 5th 5 or be s u p p r e s s e d e n t i r e l y ; by


p r o p e r choice of f i r s t t e r m s .
ify,

However, as the r e a d e r can e a s i l y v e r -

if 5 a p p e a r s once as a factor,

it r e a p p e a r s in e v e r y 5th t e r m

thereafter.
(6)

Now, as before,

let us t u r n from t h e s e two special c a s e s of 2

(the only even p r i m e ) and 5 (the only one E 5

[10) and c o n s i d e r the r e -

maining ones of the f o r m s 1 a n d . 3

We m a k e the

[10.,

following

conjectures:
(a) By p r o p e r choice of initial t e r m s we c a n m a k e any such p r i m e ,
p, f i r s t a p p e a r as a factor of any t e r m whose n u m b e r (rank)<p; or, if
p is of the form

4 k + 1, we can s u p p r e s s it a l t o g e t h e r .

(b) If such a p r i m e a p p e a r s at all, it will r e a p p e a r at the s a m e


i n t e r v a l as in the basic s e r i e s .
To t e s t t h e s e c o n j e c t u r e s ,
and 1 1 .

let us m a k e t a b l e s , a s for 5, for 7

192

Oct.

FIBONACCIOUS FACTORS
0

Jl~

6
1 |

3'-

122

[HT~

!| 2

IJ2

1 2

Note the a b s e n c e of N ' s ; since 7 is always a factor of some t e r m s of


any g e n e r a l s e r i e s .
F o r 11:

b,
Second t e r m
1

10

10

10

j] 2
|j2

10

5 i

8 j

10
5

N |
9 1

1
\\ 2
I

10

JjY

1
!

10

j| 2

10

3 J
-j-j

N
H

8
6'

10 1 N
8 | 10 !

N |
6 1

10 1
4
5 ] 9
N
6
L _ J
L_
Observing t h e s e t h r e e t a b l e s , we see the following c o m m o n f e a t u r e s :
10

8 !

(i) The top line is always a l l l ! s ;


(ii) The left column is always a l l 2 ' s , except for the top e n t r y .
(iii) One diagonal is all 3' s.
(iv) The other diagonal is all k ' s (where k will be s e e n to be the
constant of r e a p p e a r a n c e ,

in this c a s e 10), except for the upper left

corner.
(v) The nth line (except the top) is line 1 " s p a c e d out" at i n t e r vals of m from the 3.
(vi) Hence only line 1 need be computed.

1965

FIBONACCIOUS FACTORS

193

Some of the f e a t u r e s a r e obvious:


(i) The top line of l ' s m e a n only that a (in the s e r i e s
a+ b . . . ) ~ o [p.
(ii)

a, b,

Hence the f i r s t z e r o is at the f i r s t t e r m ,

The left column of 2's is s i m i l a r l y explicable.

The e x c e p -

tion of 1 at the top left c o r n e r is b e c a u s e both a and b = o, and the


e a r l i e r of the two is a, the 1st t e r m .
The diagonal of 3 r s is due to t h e i r r e p r e s e n t i n g s e r i e s in

(iii)

which the f i r s t two t e r m s a r e

a, p - a , p.

The a ' s

vary; but p in the

3rd t e r m does not.


(iv)

The identities in the other diagonal r e p r e s e n t g e n e r a l s e r i e s

of which the f i r s t two t e r m s a r e both a (2, 2, 4 . . , 3, 3, 6 . . . ,


4, 4, 8 . . . ).

The t e r m s of each of t h e s e s e r i e s a r e those of the b a s i c

(1, 1, . . . ) each multiplied by a.


s e r i e s gave a r e m a i n d e r

Consequently if any t e r m in the b a s i c

[p it would a l s o give a r e m a i n d e r (usually

different) when multiplied by a constant.


term,

U. ,

On the other hand, if the nth

of the basic s e r i e s = o [p; so a l s o a U

E o [p. That is to

say, the e a r l i e s t z e r o r e m a i n d e r in (a, a, 2a . . . ) o c c u r s at the s a m e


t e r m , r e g a r d l e s s of the value of a.
(v) The "spacing out" of Line 1 to get the e n t r i e s in Line n of
the table is explicable s i m i l a r l y .

If x = o [p so a l s o k x E o [p

while

if x o [p so a l s o kx ^ o [p9 in the f i r s t c a s e for any value of k,


and in the second so long a s k i o [p.
This m e a n s that the o c c u r r e n c e of z e r o s in any s e r i e s (a, b,
a + b . . . ) is unchanged if each t e r m in the s e r i e s is multiplied by the
s a m e constant, k o [ p .

In other w o r d s , while n o n - z e r o

remainders

m a y v a r y , p will occur as a factor of p r e c i s e l y the s a m e t e r m s in


s e r i e s (1, 2, 3, 5 . . . ), (2, 4, 6, 10 . . . ), (3, 6, 9, 15 . . . ) e t c .

Hence

the e n t r i e s in line 1 and col. 2, line 2 and col. 4, line 3 and col. 6 of
the table m u s t be the s a m e ; and s i m i l a r r e a s o n i n g shows how the r e s t
of the spacing out follows the s a m e pattern
(vi) Finally we m u s t c o n s i d e r line " 1 " of the t a b l e .
out the h a r d and obvious way r e q u i r e s us to r u n out, r e d u c e d

To fill it
[p, the

v a r i o u s s e r i e s (1, 2, 3, 5 . . . ), (1, 3, 4, 7 . . . ), (1, 4, 5, 9 . . . ) until we r e a c h a z e r o in each; and then m a k e c o r r e s p o n d i n g e n t r i e s in

194

FIBONACCIOUS FACTORS

line 1.

Oct.

This done, spacing out a s p e r (v) will compLete the t a b l e .

An a l t e r n a t i v e ,

or a c r o s s - c h e c k can be m a d e as follows:

Sup-

pose we run out the basic s e r i e s for a p r i m e we have not yet c o n s i d e r e d ,


13*

The s e r i e s r e d u c e d

[13 to the f i r s t z e r o is 1, 1, 2, 3, 5, 8, 0.

Attached is a table p a r t i a l l y filled in, with the i n v a r i a b l e 1st row


left column of 2 ' s , diagonal of 3 f s, and diagonal of 7's (the

of l ' s ,

z e r o p e r i o d of the basic s e r i e s ) .

There a r e other e n t r i e s , which we

now explain.
For

[13
R e m a i n d e r of Second T e r m (b)
0

1
8
12
5

8
12
5

3
11
10

3
11
10
2

5
1
8
12

8
12
5
1

0 marked
0
"
0
"
0 "

10

11

12

///
\\\
^ ^
UN

The e n t r y in (1, 1) is 7; b e c a u s e we have just s e e n that 7 is the


z e r o - p e r i o d of the basic s e r i e s .

T h e r e is s i m i l a r l y a 6 in the s q u a r e

(1, 2) because after a look at the b a s i c s e r i e s , we see that if we s t a r t


a n e w s e r i e s with f i r s t t e r m s 1, 2, instead of 1, 1; we a r r i v e at 0 after
6 t e r m s i n s t e a d of 7.
already,

In fact, as the 7 and 11 tables have i l l u s t r a t e d

the e n t r y in s q u a r e (1, 2) of the table is always

k - 1 , where

1965

FIBONACCIOUS FACTORS

195

k is the n u m b e r of the f i r s t z e r o t e r m in the b a s i c s e r i e s .

Similarly

t h e e n t r y i n the s q u a r e (2, 3) is always k - 2 ; and in the s q u a r e (3, 5) it


is k - 3 ; e t c . ; because as we select l a t e r and l a t e r p a i r s of t e r m s in the
basic s e r i e s to s t a r t new s e r i e s , we r e d u c e one by one the n u m b e r of
the f i r s t t e r m in which z e r o a p p e a r s .

Hence we can, without further

computation than the basic s e r i e s reduced

[p, fill in a n u m b e r of en-

t r i e s on v a r i o u s lines of the z e r o a p p e a r a n c e table (see the attached


figure for 1 3).
M o r e o v e r , we can use t h e s e e n t r i e s , with a little m o r e t r i a l and
e r r o r , to w o r k back to values in line 1 of the t a b l e .
us again look at the 13 t a b l e .

F o r e x a m p l e , let

The period of z e r o - a p p e a r a n c e s being 7

(as we have seen from the b a s i c s e r i e s ) and 3, 5 being the 4th and 5th
t e r m s in the basic s e r i e s , we know that 0 a p p e a r s at the (7-3)th t e r m
in a new s e r i e s (3, 5, 8, 0 . . . ),

Suppose we multiply the new s e r i e s ,

t e r m by t e r m , by such a factor (9) as m a k e s a still newer s e r i e s with


the f i r s t t e r m 1.
We have

3 x 9, 5 x 9, 8 x 9, Ox 9 - - -

[13

or

27 ,

45,

72,

0 ---

[13

or

1 ,

6,

7,

0 ---

[13

Hence from the e n t r y of 4 in s q u a r e (3, 5) we can check the s a m e ent r y in (1, 6); both m u s t be and a r e 4.
H e r e we note an i n t e r e s t i n g point.

Still working with modulus

13, we have the b a s i c s e r i e s


1, 1, 2, 3, 5, 8, 8, f i r s t z e r o 7
from which we get

1, 2, 3, 5, 8, 0,

zero 6

2, 3, 5, 8, 0
3, 5, 8, 0

zero 5
zero 4

5, 8

zero 3

We have found the e n t r y in the table (first zero) for 3, 5 was the s a m e
as for (1, 6).
(1, 1).

S i m i l a r l y we have seen that (1, 2) is simply 1 l e s s than

Again (2, 3) E (14, 21) 5 (1, 8); and (5, 8) = (40, 64) = (1, 12).

However, this gives us e n t r i e s in line 1 only for c l a i m s 1, 2, 6, 8 and


12.

We have no data for the r e m a i n i n g c o l u m n s , i . e . for s e r i e s b e -

ginning (1, 3) (1, 4) (1, 5) (1, 7) (1, 9) (1, 10) and (1, 11).

196

FIBONACCIOUS FACTORS

Oct.

One might at first imagine that these deficiencies were due to the
fact that we had only run our basic series out to the first zero, instead
of continuing beyond this restricted period to the full period, when not
only zero but all remainders
11,

1, 12, 0,

ever,
(3,

[13 repeat:

1, 1, 2, 3, 5, 8, 0, 8, 8, 3,

12, 12, 11, 10, 8, 5, 0, 5, 5, 10, 2, 12, 1, 0.

How-

the reader will find that the new entries in squares (8, 8) (8, 3)

11) etc. still "run back" to the same set of 5 entries on line 10
There are no entries on line 1 in columns 3, 4, 5, 7, 9, 10 and

11;

because series with first terms 1 and second terms 3, 4, 5, 7, 9,

10 and 11 have no terms divisible by 13J Recall our test, of whether


p could be a factor of a series beginning (1, b, 1 + b),
2
b - b ~1 a r e s i d u e of p ? It w i l l b e found t h a t

( 3 2 - - 3 - 1) E 5 o r 8

( 4 2 - - 4 - 1) ~ 1 1 or 2

( 5 2 -- 5 - 1) = 19= 6 or 7

( 7

(9

i. e. 9

is

7 - 1) = .41= 2 or 11

- - 9 - 1) = .71 = 6 or 7

(102 - 10 - 1) =89 = 11 or 2
( l l 2 - 11 - 1) = 109=5 or 8
are none of them residues

[130

Consequently there must be entries of N (for "never") in each


of Columns 3, 4, 5, 7, 9, 10 and 11 of Line 1.
TO SUMMARIZE as to the appearances of p as a factor of terms
in a general series (a, b, a *b).
If p is prime
2
(i) It will never appear unless =b (b

2
- ab - a ) is a residue of p.

(ii) If it can appear per (i), and does so, it will reappear at the
same interval as in the basic series.
(iii)

To determine the place of first appearance there is no sim-

pler method known to the writer than to reduce


run the series out to the first zero.

a and b [p and then

However, this can be quite a bit

simpler than running out the series, itself.

E . g . , what, if any, terms

1965

FIBONACCIOUS FACTORS

197

a r e divisible by 19 in the s e r i e s 119, 231, 350, 581?


119 = 5

[19

231 = 3

Note

[19

Hence the f i r s t 3 t e r m s = 5, 3, 8 and 3 2 - 5. 8 = -31 = 112 = 7, a r e s idue; so that 1 9 is a possible factor, then we have 5, 3, 8, 11, .0.
the 5th t e r m 931 is so d i v i s i b l e .

I.e.,

M o r e o v e r , since the z e r o period of

the basic s e r i e s is 18; this is a l s o the period in our given s e r i e s ; and


the 23rd, 41st and e v e r y 18th t e r m t h e r e a f t e r is divisible by 19.
If p is c o m p o s i t e , the r u l e s for z e r o a p p e a r a n c e s can be d e r i v e d
from the r u l e s of its p r i m e f a c t o r s in a m a n n e r e a s i l y i l l u s t r a t e d by
two e x a m p l e s :
(1) What, if any, t e r m s a r e divisible by 143 in the s e r i e s
1, 6, 7, 13 - - - ?
Since 143 = 11 x 1 3 we f i r s t check p o s s i b i l i t y of both p r i m e s as f a c t o r s
6 2 - 6 - 1 = 29 ~ 7 [11 and 3 [13
- 7 = 4 is a r e s i d u e of 11; and 3 is a r e s i d u e of 13.
Hence both p r i m e s a r e p o s s i b l e f a c t o r s
Moreover,

it can e a s i l y be found that z e r o

[l 1 a p p e a r s at the

6th t e r m with a p e r i o d of 10; while z e r o [13 a p p e a r s at the 4th t e r m


with a p e r i o d of 7.
Hence the n u m b e r

n, of the first t e r m divisible by 143 m u s t s a t -

isfy the c o n g r u e n c e s .
n^6

[10

and n = 4

[l 3

The m i n i m u m solution is 56.

Hence the 56th t e r m is the s m a l l e s t d i -

visible as r e q u i r e d by 143.
(2) On the other hand, t h e r e a r e c a s e s in which, while t h e r e m a y
be t e r m s of a s e r i e s divisible by each of two (or m o r e ) p r i m e s , t h e r e
m a y be none divisible by both (or all).

Consider

1, 7, 8, 15, 23
As the r e a d e r can check, the 4th t e r m and e v e r y 5th t h e r e a f t e r is d i visible by 5; while the 8th t e r m (99) and e v e r y 10th t h e r e a f t e r i s d i visible by 1 1 .

However,

t h e r e is no t e r m divisible by 55.

This is

198

FIBONACCIOUS FACTORS

Oct,

due to the fact that t h e r e is no solution to the simultaneous c o n g r u e n c e s


n = 4 [5

(a n u m b e r ending in 4 or 9)

n E 8 [ 10 (a n u m b e r ending in 8)
No number satisfies both conditions.
Thus t h e r e is no fixed and s i m p l e t e s t for divisibility of a gene r a l s e r i e s by a c o m p o s i t e n u m b e r .

One m u s t d e t e r m i n e for each p r i m e

factor of the c o m p o s i t e modulus, (i) the t e r m at which it f i r s t a p p e a r s


and (ii) the p e r i o d at which it r e a p p e a r s t h e r e a f t e r .

Then one m u s t

t e s t the c o n g r u e n c e s e x p r e s s i n g t h e s e two conditions for each p r i m e


in the c o m p o s i t e modulus; and e i t h e r solve them or find them to be
insoluble.
To complete this a n a l y s i s would r e q u i r e attack on the p r o b l e m of
z e r o a p p e a r a n c e s in both the b a s i c and g e n e r a l s e r i e s for moduli which
c
a r e p o w e r s of p r i m e s , p . However, this d i s c u s s i o n is postponed
pending publication of a proof by J. H. E Cohn that in the basic s e r i e s
no t e r m s a r e exact s q u a r e s , except U. , UT

and U'^.

Beyond this we offer only t h e s e C o n j e c t u r e s :


In the b a s i c s e r i e s
(i) If the k

term

i s the f i r s t one divisible by p,

choice of f i r s t two t e r m s ,

then the
c-1
and will not be g r e a t e r than the (p
)th

term.
(ii)

T h e r e will
2
chosen so that (b (iii) If t h e r e is
at the s a m e p e r i o d as

be no f i r s t a p p e a r a n c e , if the f i r s t t e r m s a r e
2
c
ba - a ) a r e n o n r e s i d u e s [p .
a f i r s t a p p e a r a n c e , t h e r e will be r e a p p e a r a n c e s
in the basic s e r i e s .
xxxxxxxxxxxxxxx

The Fibonacci A s s o c i a t i o n invites

Educationa]

Institutions to apply:
for A c a d e m i c Mem b e r s h i p in the A s s o c i a t i o n .
T h e m i n i m u m subIcription fee is $25 annually. (Ac a d e m i c Mem bers will r e c e i v e t w o
copies of each issue and wi 11 have t h e i r n a m e s listed in the J o u r n a l . )
L-

A GENERATING FUNCTION FOR FIBONACCI NUMBERS


R. G. BUSCHMAN
State University of New York, Buffalo, N. Y.

Since i n t e r e s t i n g identities for c e r t a i n n u m b e r t h e o r e t i c functions can be d e r i v e d from t h e i r g e n e r a t i n g functions, in p a r t i c u l a r gene r a t i n g functions for Dirichlet s e r i e s ,

the following p r o b l e m s e e m e d

to be of i n t e r e s t .
Problem:

Find a g e n e r a t i n g function

G which yields the F i b -

onacci n u m b e r s in the coefficients of a Dirichlet s e r i e s .


F i r s t we note that we m u s t w r i t e the s e r i e s in the form
00

(1)

G(s) =

2 f

n"S ,

n n
n=l
c = 1, the f 's i n c r e a s e too r a p i d l y .

since the s e r i e s d i v e r g e s for

P a r t of the goal i s , a s a r e s u l t , to find a simple e x p r e s s i o n to use for


c .
n
One a t t e m p t at the solution p r o c e e d s as follows.

Consider the

m o r e g e n e r a l difference equation,
(2)

u0, u

un+1

= aun + bun_1

(n*l)

from which we can w r i t e


U

[ z 2 (u l " Z l V ' Z?(U1 " z 2 u O ) 3/< z 2 " Z l )

with z, z ? = - b , z +z ? = a,

^ z?

Substituting into the D i r i c h l e t

s e r i e s we have
CD

(3)

X u

CO

c n.
n n

n= 1

= A(z n )
1

CO

c
n

z_ n
c

n= 1

+ A(z_)
Z

zT n"
n l

n= 1

w h e r e the function A is defined by

A^)

2
2
= (uQz1-u1z1)/(z1+b)
199

(u1"z1u0)/<z2"zl)

200

A GENERATING FUNCTION FOR FIBONACCI NUMBERS

Oct.

Since c

m u s t be c h o s e n to g u a r a n t e e the convergence of the s e r i e s


i
i
i
i
in (3), it is convenient to s e l e c t c = c a n d t h e n | c z 7 | < 1, | c z . | < 1.
n

Cd

Hence equation (3) can be w r i t t e n


00

(4)

, u

= A(zj) F(az.., s) + A(z ) F(az , s )

n=l
F(z, s) is a function d i s c u s s e d by T r u e s d e l l [2] .

where

,
Further

00

F(z, s) =

= z $ ( z , s, 1) ,

n=l
< denotes the L e r c h Zeta-function - some of the p r o p e r t i e s of

where

which a r e known [1:1.11] .

This allows the r e s u l t to be e x p r e s s e d in

various forms.
The difference equation
(2) can be r e w r i t t e n for
1
the form

2
n> v , , v ,, = acv + be v ,
0
1
n+1
n
n-1

=v

in

(n > 1) .
=

F o r the Fibonacci c a s e it is convenient to set c = l / 2 , so that the gene r a t i n g function for

f , that is
CO

G(s) =

(2" n f n ) n " S

n=l
can be w r i t t e n in the form
(5)

G(s) = ( 2 / / " 5 ) J F [ ( l + y - 5 ) / 4 , s] - F [(1 - S5)/4,

s] j .

To m a k e efficient use of this g e n e r a t i n g function one n e e d s to


have available i d e n t i t i e s involving the function F(z, s), e s p e c i a l l y such
identities as involve p r o d u c t s . Analogous to the ^-function, an infinite
p r o d u c t expansion for

F(z, s) in t e r m s of s, with fixed

z, might be

helpful.

REFERENCES
1.

A. E r d e l y i , et a l . , High T r a n s c e n d e n t a l Functions,
M c G r a w - H i l l , New York, 1953.

2.

C. A. T r u e s d e l l , "On a Function which o c c u r s in the T h e o r y of


the S t r u c t u r e of P o l y m e r s , " Ann. of Math. 46(1945), pp. 1 4 4 - 1 5 1 .
XXXXXXXXXXXXXXX

Vol.

l9

ADVANCED PROBLEMS AND SOLUTIONS


Edited by VERNER E. HOGGATT, JR.
San Jose State College, San Jose, California

Send all c o m m u n i c a t i o n s
Solutions to V e r n e r E. Hoggatt,

concerning Advanced P r o b l e m s

and

Jr. , Mathematics Department,

San

J o s e State College, San J o s e , California.

This d e p a r t m e n t e s p e c i a l l y

w e l c o m e s p r o b l e m s believed to be new or extending old r e s u l t s .

Pro-

p o s e r s should submit solutions or other information that will a s s i s t


the editor,.

To facilitate t h e i r consideration, solutions should be sub-

m i t t e d on s e p a r a t e signed sheets within two months after

publication

of the p r o b l e m s .
H - 6 1 Proposed by P. F. Byrd, San Jose State College, San Jose, California

(corrected)

Let
f

. = 0 for
n, k
f

0 < n <k-2, L

k
. = S
n, k
. l

f . , for
n-j, k

, , = 1 and
k-l,k
n > k .

Show that
1
2

^
i

n,>
^ 1 j. 1
,, ,
2
2k
n+1 i k

Hence
lim

lim

H - 6 5 Proposed by J. Wlodarski, Porz-Westhoven,

n, k

_ 1_

Federal Republic of Germany

The units digit of a positive i n t e g e r ,

M, is 9.

Take the 9 and put

it on the left of the r e m a i n i n g digits of M forming a new i n t e g e r , N,


such that N = 9M.

Find the s m a l l e s t

201

M for which this is p o s s i b l e .

202

ADVANCED PROBLEMS AND SOLUTIONS

H-66

Proposed by Douglas Lind, University of Virginia, Charlottesville,


Va., and
Raymond Whitney, Pennsylvania State University, Hazelton Campus, Hazelton,

Oct.
Pa.

Let

2 a.y , . = 0
j=o J n + J
be a l i n e a r h o m o g e n e o u s r e c u r r e n c e r e l a t i o n with c o n s t a n t
a..

coefficients

L e t t h e r o o t s of t h e a u x i l i a r y p o l y n o m i a l

2
j=0
and each root

r.

a.x
J

= 0

be

r,,

r~,

b e of m u l t i p l i c i t y

m.

(i = 1 , 2 , . . . , m ) .

..., r

Jeske (Lin-

e a r R e c u r r e n c e R e l a t i o n s - P a r t i , F i b o n a c c i Q u a r t e r l y , V o l . 1, N o . Z,,
pp. 69-74) showed that
oo

*. m . - l

r.t

y V = 2
n E r

n=0

i=l

j=0

b..t J .
1J

He a l s o s t a t e d that f r o m this we m a y obtain

z^x
(
">

m
= 2
i=l

m.-l
I

n
x

2
j=0

(i) Show t h a t (*) i s i n g e n e r a l i n c o r r e c t ,

b..nJ
1J

(ii) s t a t e u n d e r w h a t

t i o n s it y i e l d s t h e c o r r e c t r e s u l t , a n d (iii) g i v e t h e c o r r e c t
H-67

Proposed by J. W. Gootherts, Sunnyvale,

Let

B = (x B n , B , , . . .
0
1
two v e c t o r s in E u c l i d i a n n
ee ff ff ii cc ii ee nn tt ss of
of dd ee gg rr ee ee nn aa nn dd

condi-

formulation.

California

B ) and V = (F , F
,,,... F
, ) be
n
m
m+1
m+n
+ 1 space.
The B4' s a r e binomial c o tt hh ee FF , . ' s a r e c o n s e c u t i v e F i b o n a c c i
&
b
nm + i
n u m b e r s s t a r t i n g at any i n t e g e r m .

1965

ADVANCED PROBLEMS AND SOLUTIONS


Find the Limit of the angle between these vectors as

203
n approaches

infinity.

H - 6 8 Proposed by H. W. Gould, Wes t Virginia University,

Morgantown, West Virginia

Prove that
n
2
k=l
with equality only for
H-62

.
1
k

2
> =JL
n+2" 1

, n >1

n = 1,2.

Proposed by H. W. Gould, West Virginia University,


(corrected)

Find all polynomials

f(x)
r
X

f(x+l)=

and g(x), of the form

a.x,

j=o

Morgantown, West Virginia

a.

an integer

g(x) =

2
b.x,
j=0 J

b. an integer
J

such that
2 jx 2 f 3 (x+l) - (x+l) 2 g 3 (x)( + 3 j x 2 f 2 (x+l) - (x+l) 2 g 2 (x)(
+(2x+l) |xf(x+l) - (x+l)g(x)| = 0 .

H - 6 9 Proposed by M. N. S. Swamy, University of Saskatchewan,

Given the polynomials

Regina,

B (x) and b (x) defined by,

(x) = x B ,(x) + b , (x)


(n > 0)
v
n x ;
n-1
n-1x '
'
B (x) - (x + 1) B , (x) + b . (x) (n > 0)
n
n-1
n-1
'

b 0 (x)

Canada

= B0(x) = 1

204

ADVANCED PROBLEMS AND SOLUTIONS

Oct.

it is p o s s i b l e to show that,

B (x) = I ( n + r + 1 ) x r ,

and
n

b (x) = 1
0

n+r\
/n+rN

Vn-ry

It can a l s o be shown that the z e r o s of B (x) or b (x) a r e all r e a l ,


n
n
negative and distinct. The p r o b l e m is whether it is p o s s i b l e to f a c t o r ize B (x) and b (x). I have found that for the f i r s t few values of n,
n '
n '
the r e s u l t
n
B(X)
n

77

r = I x + 4 cos

(iTTl/

h o l d s . Does this r e s u l t hold good for all n?


s i m i l a r r e s u l t for b (x) ?
n

Z
Is it p o s s i b l e to find a

SOLUTIONS
FROM BEST SET OF K TO BEST SET OF K+l ?
H-42

Proposed by J. D. E. Konhauser, State College,

Pa.

A set of nine i n t e g e r s having the p r o p e r t y that no two p a i r s have


the s a m e sum is the set consisting of the nine consecutive Fibonacci
n u m b e r s , 1, 2, 3, 5, 8, 1 3, 21, 34, 55 with total sum 14Z.

Starting with 1,

and annexing at each step the s m a l l e s t positive i n t e g e r which p r o duces a set with the stated p r o p e r t y yields the set 1, 2, 3, 5, 8, 13, 21, 30,
39 with sum 122.

Is this the best r e s u l t ?

sum be found?

P a r t i a l solution by the p r o p o s e r .

Can a set with lower total

1965

ADVANCED PROBLEMS AND SOLUTIONS


Partial answer.

ll6o

205

The set 1, 2, 4, 5, 9, 14, 20, 26, 35 has total sum

For eight numbers the best set appears to be 1, 2, 3, 5, 9, 15, 20,

25 with sum 80.

Annexing the lowest possible integer to extend the set

to nine members requires annexing 38 which produces a set with sum


118.

It is not clear (to me, at least) how to progress from a best set

of k integers to a best set for


Comments by Murray Berg, Oakland,

The

set

k + 1 integers.

California

given above in the partial

solution is invalid since

1+5 = 4+2 = 6 and the problem asks for distinct sums for different pairs.
Comments by the Editor
An apparent solution summing to 118 was received but was discarded since the sum was larger than the partial solution given above.
Please resubmit if you read this.
AT LAST A SOLUTION
H-26

Proposed by L. Carlitz, Duke University

Let R. = (b ), where
Ki
rs

b
rs

(corrected)

= (^L'\)
_
KTI ti

(r, s = 1, 2, . . . . k+1). Then

show
Rn

_/ x
k ^ i

/r-lwk+l-r
k+1-r-s+j r+s-2j j - l \
F
F
J - l M s - j )Fn-l
n
n+iy

Letting R, = (a

), we evaluate

by extending the proposer' s

method of solving B-16 (Fibonacci Quarterly, Vol. 2, No. 2, pp. 155157).

Using Carlitz's notation, we may easily show by induction that

the transformation
x' =

y' = x + y
induces the transformation

206

ADVANCED PROBLEMS AND SOLUTIONS


,0, k
(0)y

> _

,1,
k-i , .1. k
(L)xy
+ ( 0 )y

,k-i ,
X

Oct.

y'

k:
x y

,k-l,

,k-l

k-l)x

k-1

, .k-1,

k-1 , , k - l .

y + . . . + l ! )xy

+(

)y

y ' k = <)xk + ( ^ J x ^ y + . . . + ( k )xy k _ 1 + ( k )y k


Carlitz also showed that the

T?- is given by

M)

(1)

,x + F y
n-1
n

F x +F
y
n
n+1'

so that T\ induces the transformation


(2)

Tj: I ^ ( k - r + l ,
k I

W,r-1

k+1
=

k+l-B s-1

We note here a misprint in the B-16 solution:


should begin
with
&

T . instead of
2
(1) into (2) to obtain

{ r = 1 ; 2>

...

T, .
1

the last transformation

To evaluate

rs

, we substitute

_,
k+l-s s-1
_ .k+l-r,_
,
xr-l
T a x
y
= s(F , x + F Jy)
(F x + F ,,y)
%
7
T
rs
'
n-1
n'
n
n+1 '
s=l
k+1 - r
, _L1
,
1-r-i.^i k + l - r - i i
k+l-r jk
.
y
v
. '
i
' n-1
n
1=0

r-1

X 2

,
, . >
,r-lx._r-l -J^J

j=0

i=o

j=o

-x

>k+1)>

rs

j = 1

)F
n

n+r
F^+1x

i
r-l-j j

n+1

1965

ADVANCED PROBLEMS AND SOLUTIONS

207

k+1 - s s -1
of x
y
, summing all terms of the last

We equate coefficients

sum with i+j = s - 1 , and since j i s - 1 we find


_
rs "

s- 1

,k+l-rur-l
k+2-r-s+j r+s-2-2j j
F
F
J0ls-l-jMj-l,Fn-l
n
n+1

_
"

j= l

. k + l - r w r - l . k+1-r-s+j
S
"J M J - 1 ) n-1

r+s-2jj-l
F
n
n+1 '

ANOTHER LATE ONE


H-38

Proposed by R. G. Buschman, SUNY, Buffalo, N. Y.

(See Fibonacci Numbers, Chebyshev Polynomials, Generaliza tions and Difference

Equations; Vol. 1, No. 4, Dec. 1963, pp. 1-7.)

Show
(u , + (-b) r u
)/u = X .
n+r
n-r 7 / n
r
Solution by Douglas

hind

Let z1 / z ? be the roots of z


b = z, z ? .

- az - b = 0, and note a = z, + z ? ,

We recall from the article that


Un = j (UJ-ZJUQJZJ - ( t l j - Z ^ J z * [ / ( Z 2 - Z l )

and

* n = I ( a "" 2 z 1 ) z 2 " ( a - 2 z 2 ) z i ' } /( Z 2" Z 1^ *


Now
1

since
U

A = z0 + z
n
2
1
a-2z,= z ? ~z ] = -(a-2z ), so that
\
nAr

\i
\ n
)(urZlU0)z2

+ r

/
vn +
" ^U1~Z2U0)Z1
-

= u , + (-b) r u
n+r
n~r
the desired result.
Also solved by Clyde Bridget and the proposer.

. / u\ r /
\ n "T
" b ) (urZlU0)z2

+ (

(-b)r(urz2u0)z^r[/(z2-Zl)

THE FIBONACCI NUMBERS AND THE "MAGIC NUMBERS


J. WLODARSKI
F e d e r a l R e p u b l i c of Germany

It was r e p o r t e d h e r e (The Fibonacci Q u a r t e r l y , i s s u e 4, 1963)


that one of the fundamental a s y m m e t r i e s in the world of a t o m s is a s y m m e t r i c a l d i s t r i b u t i o n of fission f r a g m e n t s by m a s s n u m b e r s r e s u l t i n g
from the b o m b a r d m e n t of m o s t heavy nuclei (by t h e r m a l n e u t r o n s ) .
The p r o b l e m of this type of the a s y m m e t r y is one of m o s t difficult p r o b l e m s in the b r a n c h of f i s s i o n - p h y s i c s .
It s e e m s that by the h e r e mentioned a s y m m e t r y t h e r e is a connection between the Fibonacci n u m b e r s (. . . 34, 55, 89, 144, . . . ) and
the s o - c a l l e d " m a g i c " n u m b e r s (2, 8, 20, 28, 50, 82 for p r o t o n s and
2, 8, 20, 28, 50, 82, 126 for n e u t r o n s ) , which a r e well known in
nuclear physics.
O o L

As a m a t t e r of fact the f i s s i o n - n u c l e u s Q ? U
p o s s e s s e s 144
n e u t r o n s and consequently a sufficient quantity of n e u t r o n s to form two
neutron-shells:
trons.

one with 50 n e u t r o n s and the other with 82 neu-

If the r e s t of 12 n e u t r o n s [144 - (50 + 82)] divide in two equal

p a r t s , the whole n u m b e r of n e u t r o n s in the heavy fragment is 82 + 6 =


88 (89)

and in the light f r a g m e n t s 50 + 6 = 56 (55). ^


T O /

The 92 p r o t o n s of the nucleus Q?U


can a l s o form two shells
with " m a g i c " n u m b e r s of p r o t o n s : 28 and 50 r e s p e c t i v e l y . If the r e s t
of p r o t o n s [92 - (28 + 50)] = 14 divide in the s a m e m a n n e r as the r e s t
of the 12 n e u t r o n s , the whole n u m b e r of protons in light f i s s i o n - f r a g m e n t
should be: 28 -r 7 = 35(34) and in the heavy fragment:

50 + 7 = 57(55).

These n u m b e r s of p r o t o n s (35 and 57) and the n e u t r o n s (56 and


88) in both f i s s i o n - f r a g m e n t s of the nucleus
well the m o s t e x p e r i m e n t a l r e s u l t s .

Q?U

conform r a t h e r

The n u m b e r in p a r e n t h e s i s is the n e a r e s t F i b o n a c c i n u m b e r .

1)

Mukhin,

K. N. ,

Introduction to Nuclear

USSR (1963), p . 350.

xxxxxxxxxxxxxx
208

Physics.

Moskow,

AN ELEMENTARY METHOD OF SUMMATION


D. G. MEAD
University of Santa Clara, Santa Clara, California

T h e p u r p o s e of t h i s n o t e i s t o p r e s e n t a n e l e m e n t a r y m e t h o d f o r
s u m m i n g the f i r s t

n t e r m s of a s e q u e n c e w h i c h s a t i s f i e s a g i v e n h o m o -

geneous linear r e c u r s i o n relatione

The method i s ,

in fact,

a simple

e x t e n s i o n of t h a t n o r m a l l y u s e d f o r s u m m i n g a g e o m e t r i c p r o g r e s s i o n ,
which we first r e c a l l .
Let:
S = a + ar + ar

+ . . . + ar

Then:
-rS

- ar - ar

. - ar

n+1
- ar

Therefore:
S(l - r) = a - a r
a n d if

n+1

r / 1,
n+1
S =

1 - r

We n o w t u r n to t h e g e n e r a l c a s e .
G.
J

satisfies

G . ,, + 2

(1)
w h e r e the

j+k

c.

i = 1

G.,,

. = 0,

j+k-i

a r e fixed q u a n t i t i e s , we w r i t e , a s above

S = G, + G~ + G~ + . . . . + J G, , ,
1 2
3
i k+1
t
0 , 8 = 0 , 0 , + c , G~ + . . .
+ g c , G,
1
1 1 1 2
I l k
C 2 S:

If f o r e v e r y p o s i t i v e i n t e g e r j ,

c2G1+.

+ G, , k+2

+. . . +G

|
I
i
+c,G, , , + . . . +c,G
, |+ c , G
1 k+1
1 n-1.5
1 n
n

c9G, , +c~G,
2 k-1
2 k

+ . . . +c~G
^i+c-G
,+c~G
2 n-2i
2 n-1
2 n

+ . . . + c , kG n - k, s; + . . .

v^ G ii-

+ c

vkG ?2

209

+ ck, G n

210

AN ELEMENTARY METHOD OF SUMMATION

Oct.

Since, adding v e r t i c a l l y and using (1), the sum of the t e r m s inside


the dotted lines is z e r o , we see:
S(l +c ] [ +. . + c k ) = G x (l +c ] [ +. . . + c k

) +G 2 (1 + c x +. . , + c k

) +. . . + G k

+ G (c. + c 0 +. . . + c. ) + G , (c 0 +. . . + c. ) + - . + c. G . , . .
n 1
2
k . n-1 2
k
k n-k+1
If 1 + c, + c . + , . . + c. , / 0, we can solve for
1 2
k-1
The s a m e method can be u s e d to find

S.

t
X i G- for a given t,
1
i=l
if the G. satisfy (1) To facilitate the p r e s e n t a t i o n , we collect some
t e r m i n o l o g y and facts a
Let E be the o p e r a t o r with the p r o p e r t y that
EG. = G. ,, .
I
l+l
To say that G. satisfies (1) is equivalent to the s t a t e m e n t that the
operator
v

0(E) = E K +

k
2
i=l

c. E K " X
1

when applied to any G., yields z e r o (E

being the identity operator),,

The a s s o c i a t e d polynomial
k

k
0(x) = x

X
i=l

k-i
c.x
1

is called the c h a r a c t e r i s t i c polynomial.'** The s p e c i a l r o l e of the n u m b e r one in our g e n e r a l i z a t i o n is now e a s i l y stated, for
1 + c, + . . . + c, ^ 0
1
k '
if and only if unity is not a root of the c h a r a c t e r i s t i c polynomial.
0(x) is unique if we a s s u m e 0 ( E ) G. = 0 for all positive j
the d e g r e e of 0 (x) > k.

implies

1965

AN ELEMENTARY METHOD OF SUMMATION

It is known ( [2] , pp. 548-552) that if 0 ( E ) G . = 0, then


J
satisfies
[0(E)]

B. = 0, for

211
B^-j^G.
J J
j

t > 1.

If 0(1) / 0 then if,(l) ^ 0, w h e r e i//(x) = [0(x)] , and the method j u s t


d e s c r i b e d can be used to find
n

T = 2
j=l

t-1
B. = 2 j
J
j=l

G.
J

Writing

+2
d x "1,
i=l

0(x) = x
we find:

pnT =

kt-j
kt-1
I
P-B. + X
r.B

where
kt-j
p. = 1 + X d. and r . =
x
J
i=i
J

kt

i=j+i

d. .
x

Since 0 ( E ) G . = 0 and B. = j G.? one can e a s i l y obtain T in t e r m s of


1

k-1
n-k+2
n
The a s s u m p t i o n that unity not be a root of the c h a r a c t e r i s t i c
polynomial h a s been c r i t i c a l to our d i s c u s s i o n so far. We now a s s u m e
\G.{

satisfies
X(E) G. = 0
J

w h e r e X(E) is a polynomial with X(l) = 0o

F a c t o r i n g out all the fac-

t o r s x - 1 in X(x), we obtain
X(x) = (x - l ) a 0(x), w h e r e

0(1) 4 0.

212

AN ELEMENTARY METHOD OF SUMMATION

Letting

Oct.

C. = </>(E)G.3 we note:

6(1)S = <t>Q . ^

n-k
G.q. + .^

G. = . ^

k-1
C. + ^ G

. s.

where
v
X(x) = x* +

i=l

k-j
q. = 1 + X
J
i=l

v
K_1
c..x
1

c. and

s. =

k
2
i=j+L

c.
*

However, it is known ( [ 2 ] , pp. 548-552) that if (E - l ) a C. = 0,


then C. is a polynomial of d e g r e e < a ~ . l . If we a s s u m e the f o r m u l a s
for
n
j=l
a r e known, for

j a positive i n t e g e r , the only p r o b l e m r e m a i n i n g is


a-1
that of d e t e r m i n i n g the polynomial C. = d_ + d. j + . . . + d . j
. It

j
0
1
a-1
is e a s y to show that the difference o p e r a t o r E - l when applied to a
polynomial of d e g r e e r yields a polynomial of d e g r e e r - 1. T h e r e fore (E - 1) C-. involves only d . , d 7 . . . , d. and the s y s t e m of
l i n e a r equations on the d. obtained by computing (E - 1) J C-., j = 0, 1,
2, . . . , a - 1 can c l e a r l y be solved for the d..
The above is a g e n e r a l i z a t i o n of the technique used by E r b a c h e r
and Fuchs to solve p r o b l e m H - 1 7 . [4 J
Example:

A s s u m e that for

X(E)G. = 0, w h e r e

X(x) = (x - I )

GL = G 2 = G 3 = G 4 = G
*G 2 = 0,
(E - 1 )
Hence

(x

- 3x

satisfies
3

+ 4x + 2) = ( x - 1 ) <#(x). If

= 0, G 6 = 1, then C 1 = 0 ( E ) G 1 = 0, C 2 = 0(E)

C 3 = 0(E) G 3 = 1.

e a c h positive i n t e g e r j , G.

With C. = dQ + d } j + d 2 j 2 ,

C 1 = 2d 2 = 1, (E - 1) CL = dl + 3d 2 = 0 and C
2

C- = 1 - (3/2) j + j / 2 and

we find

= dQ + c^ + o^.

1965

AN E L E M E N T A R Y METHOD OF SUMMATION

n-3
< ( 1 ) S = 4 S = 2 G ,i - 2 G L9 + G .5 + .1 ^ (1 - ( 3 'j ) / 2 + j

213

/' 2 ) + 3 G n + 6 G n-1i + 2 G n - 2
0

n-3

(1 - (3j)/2 + f/Z)

In conclusion,

+ 3 0 ^ + 6G

, + 2G^

we have seen how the e l e m e n t a r y method used to

sum a g e o m e t r i c p r o g r e s s i o n can be g e n e r a l i z e d to find the sum of the


first

n t e r m s of a sequence "which satisfies a linear homogeneous r e -

c u r s i o n reLation.

It m a y be w o r t h stating that this method is applicable

to a sequence whose t e r m s a r e p r o d u c t s of c o r r e s p o n d i n g t e r m s

of

s e q u e n c e s each of which satisfy a linear homogeneous r e c u r s i o n r e l a tion (see [ l ] pp. 42-45 for a s p e c i a l c a s e ) .
We p r o p o s e as a p r o b l e m for the r e a d e r :
the sum of the f i r s t

Find in closed form

n t e r m s of the sequence Jw { :
1, 2, 10, 36, 145, . . .

where w ' = F G with F ^ = F ,. + F (F, = F 9 = 1) and


n
n n
n+2
n+1
n 1 &
G n+2
, ? = 2Gn+1,. + Gn (G11 = 1, G29 = 2).REFERENCES
1.

Dov J a r d e n , R e c u r r i n g Sequences, J e r u s a l e m , 1958.

2.

C. J o r d a n ,

"Calculus of Finite Differences, " Chelsea,

New

York, Ed. 1950.


3.

J a m e s A. J e s k e ,

" L i n e a r R e c u r r e n c e Relations, P a r t i , " The

Fibonacci Q u a r t e r l y , Vol. 1, No. 2, pp. 69-74.


4.

P r o b l e m H-17,

The Fibonacci Q u a r t e r l y ,

P r o p o s e d in Vol. 1,

No. 2, 1963, p. 55 and solved by Joseph E r b a c h e r and John Allen


Fuchs in Vol. 2, No. 1, 1964, p. 5 1 .

xxxxxxxxxxxxxxx

ON IDENTITIES INVOLVING FIBONACCI NUMBERS


V. C. HARRIS
San Diego State College, San Diego, California

R a t h e r extensive l i s t s of identities involving Fibonacci n u m b e r s


have been given by K. Subba Rao [l] and by David Zeitlin [2] .

Addi-

tional identities a r e p r e s e n t e d h e r e , with the feature that s u m m a t i o n


by p a r t s h a s been used for effecting the proofs (except for identity

3).

Let f o = 0 and f,1 = 1 and let f n =f n-1, + f n-Z, for

Then

&

(2)

n > 2.

2 kfk = nf n + 2 - fn+3 + 2
k=0

(D k kf k = l - l l V ) ^ . ! + l-D*'1*^

1
k=0

<3>

- Z>

(-i)kf2k=[(-Dn(f2n+2+2n)-i]/5

2
k=0

(4)

(-l)k2k+l='[<-1)n<f2n+3+f2a+l>+2]/5

I
k=u

<5)

(6)

(7)

(8)

*
k=0

*
k=0

kf

kf

2k=(n+1)f2n+l

2k+l

1
(-D^k
k=0

-f2n+2

(n+1)f2n+2-f2n+3+1

^ ^ Z n + Z

<-D k kf 2 k+l = t - D ^ Z n + S
k=0
214

< n + 1 > f 2n>/ 5

< n + 1 > f 2n+l>/ 5 "

'b

1965

ON I D E N T I T I E S I N V O L V I N G
n

(9)

2,
kf

2
k=0

(10)

X
k=0
n

(12)
* '

X
i n
k=0

,2
=(n+2)f

2 n + 1

-(2n

l)f

-2

2 n

k 2 f 2 k + 1 = ( n 2 + 2 ) f 2 n + 2 - (2n+l)f

2
k=0

t")

2 k

215

- 1

4=<2n+1>f2nf2n+l-4+l+1

k
(-1) kf

n
^
= v(-1) x( n + l ) f
, ,,
m+3k
'
' m+3n+l
+f
2
- vv
( ( - l )' n f m +.-a
3 n +.29 m - li )7 // > m = 2 3 3 , .

(13)
A
'

n
X
in
k=0

,
(-l)Kkf

,., = (-l)n(n+l)f
,
,
m+4k x ' x
m + 4A n + 2
-((-l)nfm+4n+4 + f

(14)

121

)A

m=2.3,...

(-l)kkfm+5kM-Dn[(55n+35)fm+5n+1

2
k=0

-25fm-5n+2+(22n+18)W5n]
-[
n
2
k=0

(15)

(16)

n
2
k=0

(17)

(18)

K
2
kx=0
2
k=0

2
k=0

k \

2 0 f

m+l-

1 7 f

m-

1 0 f

m-l]-

m = 1

'2'

k
2 :f.
= f n+4 - (n+3)
n
k =0 K l

= (n+Df+4

- f

+6

+ 5 - n(n+l)/2

l
= ( n 2 + 2 ) f n + 2 -. ( 2 n - 3 ) f n + 3

k 3 f k = ( n 3 + 6n - 12)f

+2

- 8

- ( 3 n 2 - 9n + 1 9 ) f n + 3 + 50

216

FIBONACCI NUMBERS

(19)

2
k 4 f, = (n 4 + 12n 2 - 48n + 98)f
k
n
k=0
^

Oct.

- ( 4 n 3 - 18n 2 + 76n - 159)f n + 3 - 416

(20)

k=o

(21)

,S n
k=0
n

(22)

n n+1

2
_ 1
k f k+l = 2 f n+2 f n+l f n

f f i , , = i (f , , f , , f - (-1)
, k k+2 2 n+3 n+1 n
k=0

(23)

2
v n

f .+1)
n-1

J i<+ln^-1>nn-l+1^

k=u

5 kfk ^EiKtiV^'Vi'-iW:n+1
k=0

{24)

+ (~l) n (3f - 2f , ) + 5
n
n _ 1
^ ~
4
The well-known method of s u m m a t i o n by p a r t s i s e s t a b l i s h e d from
the identity
u k A v k = A (u k v k ) - v k + 1 A u k
On s u m m i n g t h e r e r e s u l t s

k=0

Of c o u r s e ,

u, A v, = u. v,
k
k
kk

n+1
0

n
"

k=0

a suitable choice of u, and A v,

in i n t e g r a t i o n by p a r t s .

kk++l i

A u

kk

i s e s s e n t i a l j u s t a s it i s

In o r d e r to find v ^ from Av]j r e s u l t s i n [ i j

1965

ON IDENTITIES INVOLVING

and [2] have been used when needed.

Also,

217

any constant term in v,

can be omitted in the two t e r m s of the right member.


To prove (1), let u ' = k and A v
k-1
1

v, =
k

Omitting the constant

n+1
1

kf

k=0

kf

k+i

v, ,

Then A u , = 1 and

we find

" k=0
*

n0

k + 1

f. = f. x l -1
1
k+1

i=Q

-1 from

=f .

k+Z = < n + 1 f n t Z - W 1 - V

nf n+2
, 0 - x(fn+4
, - fn+2
x o ); + 2
= nf

n+2 " fn+3

+ 2

To prove (2), let u , = k and

V ( - 1 ) \K=

k=0

. k - 1

i=o

(-D'f. -

2 (-1)%x

i=o

Then Au, = 1
with k > 2

x
and v, = (-1) f,
k - 2? - l .

(-l)kkfk = I

k=2

Omitting the t e r m

(~l)kkfv - 1 = M - l ^ f .k-2
n_1

( - D V D ^ . !

+ 1
k=l

= (-l) n (n+l)f

n
2
k=2

(-D

- 2

n-2

To prove (3) and (4), together, write in (3) u, = (-1)


k
Av

k=

1=0

\ - 1

k-1

hi'

i=0

2i

v.,

,
( - i ) \ .

+ (~l)n~l
n-1

n+1

-1 from

and

218

FIBONACCI NUMBERS
k-1

So that A u k = 2 ( - l ) "

and v k =

f 2k ^

A= X (-D k f 2k - S ( - ^ z k ^ - ^ z k
Zk

k=0
=

Zk

k=l

Then
n+1

Zk

Oct.

"i

n
- 2 X (-l)k+1f2k+1 " ^
k=0
^k+i

^'^zmr^^

where
B=

X (-1) f
2k + r
k=0

In (4) let u k = ( - l ) k and A v k = f 2 k + 1


Then

so A U R = 2 ( - l ) k + 1

n+1
B

1 kf

? 0 <" ) 2k + l = (" ) 2k

- 2

1 kf

, .

( - i r + 1 f .2k+2

X
k=0

^ ^ W z + ^ ^ Z n + Z - 2A

Solving gives the r e s u l t s .


To obtain (5) let u, = k and then v, = f-,
n
2
k=0

and vfe= f^.

n+1
kf

2k=kf2k-l

n
X
2

k=0

2k+l

,.

= (n+1)f

This gives

2n+l

"f2n+2

The o t h e r s a r e proved s i m i l a r l y , except that (23) w a s obtained


from (21) and (22). Note that the s a m e method could be used to extend
the r e s u l t s .

REFERENCES
K0 Subba Rao, "Some P r o p e r t i e s of Fibonacci N u m b e r s , " A m e r i c a n M a t h e m a t i c a l Monthly, Vol. 60, 1953, pp. 680-684.
David Zeitlin, "On Identities for Fibonacci N u m b e r s , " A m e r i c a n
M a t h e m a t i c a l Monthly, Vol. 70, 1963, pp. 9 8 7 - 9 9 1 .
XXXXXXXXXXXXXXX

CONCERNING THE EUCLIDEAN ALGORITHM


R. P. K E L I S K Y
IBM Watson Research Center, Yorktown Heights, New York

In m o s t d i s c u s s i o n s of the i n t e g e r solutions of the equation


(1)

ax + by = 1,

(a, b) = 1,

r e f e r e n c e is m a d e to the fact that an i n t e g e r solution of (1) m a y be obtained by using the Euclidean a l g o r i t h m .

With the r e s t r i c t i o n that

a > b > 1 we shall show that in the x - y plane the solution of (1) obtained by the E u c l i d e a n a l g o r i t h m is the lattice point on the line (1) which
is n e a r e s t the origin.

This is p r o b a b l y not a new r e s u l t , but we cannot

find a r e f e r e n c e to it in the literature,,

Dickson [l, pp. 41-521 gives

other a l g o r i t h m s for solving (!) for which it is known that the a l g o r i t h m


yields the lattice point on (1) which is n e a r e s t the origin.
Suppose a > b, (a, b) = 1, and a ^ 1 (mod b) and c o n s i d e r the
Euclidean a l g o r i t h m applied to the i n t e g e r s

a and b. One obtains the

well-known sequence of equations:


a

= b

n-2

n-3

= r
n-2

ll

1 < r1

<

^2

+ r2,

1 < r2

<

rx

q3

1 < r

<

^n-1

q
n-1

Ty

n-1

, < r 9
n-1
n-2

+ r
n

with r = 1.
The r e q u i r e m e n t that a ^ 1 (mod b) is equivalent to
r > 1, and hence the E u c l i d e a n a l g o r i t h m will r e q u i r e at l e a s t a s e c ond stepo Hence n ^ 2 and r . > 2
^
n-1 ~
To obtain a solution of (1) one then d e r i v e s the following sequence
of equations in which, for notational convenience, a = r ,
219

and

b = r

CONCERNING THE EUCLIDEAN ALGORITHM

220

1 = r

~
n

(3)

Oct.

~ - q r 1
n-Z
n n-1
- ^n
q r n - 3Q + ( l + q^nq ^ n - 1^' r n - 2

= P . r . . + Q. r .
1 n-i-1
l n-i

= P

r .
+ Q rn .
n - 1
n 0
The P . and Q. a r e polynomials in the q. and the solution (P , Q )
l
l r J
n
n n
will be called the Euclidean a l g o r i t h m solution of (1)0 It is d e t e r m i n e d
uniquely by the a l g o r i t h m d e s c r i b e d by the equations (2) and (3).
L e m m a 1:
| p I < 4 b and | Q I < I a .
1
n'
2
' n'
2
Proof:

(4)

We f i r s t p r o v e by induction

|p. | < I2 r n -.i

and
(5)

|Q.| < 2 r n _ i - l

for

i=

with equality p o s s i b l e in (4) only if i = 1.


1 = P. r
I

l-.-n,
We have

. . + Q. r
n-i-1
l n-i

and since
r n - i.- 20 = r n - i- 1n q^ n - i. + r n - i
it follows that
1 = Q.I r n - i.- 29 + x(P.I - ^qn - i.Q.)r
r n - i. - 11
and we have the r e c u r r e n c e r e l a t i o n s
(6)
and

P.+1

= Q.

1965

CONCERNING THE EUCLIDEAN ALGORITHM

(7)

= P. - q .Q.

l+l

with P . = 1 and Q_ = -q .
1
1
^n
that r , > 2. Similarly,

^n-i i

To prove that
^
'n-2

Q,

<.

V i

From (6) it follows that

221

| p . I = 1 % i- r , rec a l l
' 1'
2 n-1

n-2 <
^ 1
r
,
2 n-2
n-1

| p | < I r ' , and from (7) | Q | < I r


2 n-2'
2 n-3

since
Q,

lQl

P ,1 - <HLn. -, lQ~,l I > |Pi I +q.n-1


< ^ - r

n + q , * r

n-1

^n-1

0
n-2

2 rn-3
Now suppose that
lpkl<2rn-k
for

k = 2, . . . . , i.

a n d

lQkl<7rn-k-l

Then from (6)


|p,
'
k+1

'Q

kl

<

7rn-k.l'

and
Q k+1

|p

<

k~ V-kQkl=

kl

< i2rx n-k

+ q

2 rn-k-2

This

completes

the induction.

proved the lemma if we take

Since

Vkl

n-k ( Z r n-k-l )

r , = a and r 0 = b, we have

i = n in (4) and (5).

It seems intuitively clear that there cannot be two lattice points


on (1) which a r e equidistant from the origin if a / b. The proof of this
is straightforward but for completeness we give it here.

222

CONCERNING THE EUCLIDEAN ALGORITHM

L e m m a 2:

Oct.

If a > b > 0 and (a, b) = 1, t h e r e do not exist two d i s -

tinct lattice points on ax + by = 1 which a r e equidistant from the origin.


Proof:

Suppose

(a, (3) and (<f, */) a r e distinct lattice

on the given line which a r e equidistant from the origin.

points

Then

a 2 + p 2 = f 2 + 1Z

(8)

and act + bp = a f + b*7 = 1. We solve for

p in t e r m s of a, r\

in t e r m s

of , and substitute t h e s e in (8) to obtain


(a 2 - f 2 ) b 2 = 2a(a - f ) - a 2 ( a 2 - 2 ) .

(9)
Since a / f

by h y p o t h e s i s ,
(a + ) b 2 = 2a - a 2 ( a + f ) .

(10)

But this i m p l i e s that a | ( a + f ) s i n c e (a, b) = 1, and a l s o that (a + f ) ) 2 a .


Hence, a + <f = a, or a + <f = 2a. If a + <jf = a, then (10) i m p l i e s
2
2
the Diophantine equation a + b = 2 which is i m p o s s i b l e if a / b.
If a + = 2a, then a + b = 1. C l e a r l y t h e r e is no solution to
this equation such that a ^ b ^ 0 and (a, b) = 1.
It is well known that if (x , y n ) is any lattice point on (1) then all
of the lattice points on (1) a r e given by the equations
x = x n - bt
y = y0
where

at

t runs over the set of all i n t e g e r s .


Theorem.

We can now prove our

If a > b > 1 and (a, b) = 1 then the Euclidean a l -

gorithm solution of (1) is the lattice point on (1) which is n e a r e s t the


origin.
Proof.

F i r s t suppose that a ^ l

(mod b).

Denote the E u c l i d e a n

a l g o r i t h m solution of (1) by (P , Q ). C l e a r l y the set, S, of positive


2
o11 n
2
2
i n t e &g e r s x(P - bt) + x(Q + at)^ h a s a s m a l l e s t m e m b e r . If P + Q
n
n
n
n
i s not the s m a l l e s t n u m b e r in S then t h e r e e x i s t s an i n t e g e r t ^ 0 such
that
P 2 + Q 2 > (P - bt) 2 + (Q + a t ) 2
n
n
n
n

1965

CONCERNING THE EUCLIDEAN ALGORITHM

223

or
0< (aZ + b2) Itl < 2 | p b - Q a l .
1
J
'
n
n j
But from the lemma we have
0 < (a 2 +b 2 ) | t | < 2 ( | p
j

This is impossible; hence

| b + |Q |a)< a 2 + b 2 .
n

'

n'

t = 0 and (P , Q ) is the smallest number


n n

in S.
The only remaining case is if a = 1 (mod b) and a > b ^ 1. Here
the Euclidean algorithm is complete in one step and P . = 1 and
Q, = - q, = - (a - l ) / b .
can be rewritten

S(t) = (P, - bt) 2 + (Q

The expression
,-

c l t

c-a |

I F

+ at)

.
,1

7i
b

2
where

c =a

2
+b .

Now S(t) is a minimum for

b > 1 and c^ a imply that


fore,

the integer

c(b-l) + a > 0,

or

t=t*=(c-a)/bc,
0 < t- <: 10

t for which S(t) is a minimum is either

It is easy to show that

S(l) > S(0) if

(c-a)/bc < 1/2.

but

There0 or

1.

But

be < Ib and b > 1;


hence

(P, , Q, ) is the point on ax + by = 1 whichis nearest the origin.

This completes the proof of the theorem.


It is an easy consequence of this theorem that if a and b are
consecutive Fibonacci numbers, a > b > 1, then the Lattice point

P on

the Line ax + by = 1 which is nearest the origin has Fibonacci coordinates.

In fact,

if a = F

. , , then P is (F , , - F ) where n is
m+1
n-1
lithe greatest even integer not exceeding m. This foLlows readily from
the identity
F

,, F . - F F = (-l)nF
...
m+1 n-1
m n
m-n+1
REFERENCES

1.

Dickson,

L. E. ,

"History of the Theory of Numbers, " Vol. 2,

Chelsea, New York (1952).

xxxxxxxxxxxxxxx

A STRIP METHOD OF SUMMING LINEAR FIBONACCI EXPRESSIONS


BROTHER U. ALFRED
St. Mary's College, California

Given a l i n e a r F i b o n a c c i e x p r e s s i o n such a s
362880 F , , . - 2177280 F ^ n + 5594400 F J_. n - 8013600 F 1 1 C
r+21
r+19
r+17
r+15
+ 6972840 F ,. . - 3759840 F . . . +1225230 F 1 0 - 223290 F tn
r+13
r+11
r+9
r+7
+ 19171 F , c - 512 F , , + F _,_.
r+5
r+3
r+1
we w i s h to e x p r e s s t h i s , for e x a m p l e , as
A F ,, , + B F . . .
r+11
r+10
The f o r m u l a s for doing so a r e well known being
F

= F
n

and
F

However,

F
k+1

+F
n-k

F
k

n + k + l " F k+1

n-k-1
F

n+k

the d i r e c t p r o c e s s can be r e p l a c e d by a s t r i p method in

which the given coefficients a r e a r r a n g e d in descending o r d e r of


subscripts,

one space being allowed for each s u b s c r i p t , even though

c e r t a i n s u b s c r i p t s m a y be m i s s i n g in the given l i n e a r e x p r e s s i o n .
This m a y be done conveniently on ruled paper,

the s t r i p employed

having the s a m e spacing in its rulings as the p a p e r .


The s t r i p c o n s i s t s of the Fibonacci n u m b e r s in descending o r d e r .
To obtain the coefficient of the higher s u b s c r i p t F i b o n a c c i n u m b e r in
the s u m m a t i o n ,
subscript,

place the 1 above the z e r o at the place of the h i g h e r

multiply each n u m b e r on the s t r i p by the c o r r e s p o n d i n g

given coefficient and add the r e s u l t s .


lower s u b s c r i p t F i b o n a c c i n u m b e r ,

To find the coefficient of the

do likewise with the 1 below the

z e r o opposite the position of the lower s u b s c r i p t Fibonacci n u m b e r .


The w o r k is shown for the example given at the beginning of
this note.
224

1965

A STRIP METHOD OF SUMMING LINEAR

225

U P P E R SUBSCRIPT CALCULATION
STRIP
89
55
34

21
13
8
5
3
2
1
1
0
1
-1
2
-3
5
-8
13
-21
34
STRIP
55
34
21
13
8
5
3
2
1
1
0
1
-1
2
-3
5
-8
13
-21
34
-55

GIVEN COEFFICIENTS

PRODUCTS

362880

32296320

-2177280

-74027520

5594400

72727200

-8013600

-40068000

6972840

13945680

-3759840 ( F r + U )

-3759840

1225230

1225230

-223290

-446580

19171

95855

-512

-6656

34
1981723

LOWER SUBSCRIPT CALCULATIONS


GIVEN COEFFICIENTS

PRODUCTS

362880

19958400

-2177280

-45722880

5594400

44755200

-8013600

-24040800

6972840

6972840

-3759840 (F

r+10'

1225230

-1225230

-223290

669870

19171

-153368

-512

10752

-55
1224729

226

FIBONACCI EXPRESSIONS

Oct.

The final r e s u l t would thus be


1981723 F . . . + 1224729 F ,. n
r+11
r+10
In c a r r y i n g out t h e s e calculations it goes without saying that the p r o d ucts need not be w r i t t e n out but m a y be cumulated on a c a l c u l a t o r .
XXXXXXXXXXXXXXX

THE FIBONACCI ASSOCIATION ANNOUNCES


The a p p e a r a n c e of a booklet entitled: "Introduction to Fibonacci D i s c o v e r y " by B r o t h e r U. Alfred, Managing Editor of the Fibonacci
Q u a r t e r lyk As the title i m p l i e s the aim of this publication is to p r o vide the r e a d e r with the opportunity to w o r k out v a r i o u s facets of the
Fibonacci n u m b e r s by himself. At the s a m e t i m e , t h e r e is sufficient
help in the form of a n s w e r s and explanations to r e a s s u r e him r e g a r d ing the c o r r e c t n e s s of his work.
The t r e a t m e n t is r e l a t i v e l y brief, t h e r e being some sixty pages in all.
The m a t e r i a l was set up by t y p e w r i t e r and subsequently lithographed.
The books have a paper cover and a r e held together by glue binding.
P r i c e per copy is $1.50 with a quantity p r i c e of $1. 25 when four or
m o r e copies a r e o r d e r e d at once. The following topics a r e t r e a t e d :
D i s c o v e r i n g Fibonacci F o r m u l a s
Proof of F o r m u l a s by M a t h e m a t i c a l Induction
The Fibonacci Shift F o r m u l a s
Explicit F o r m u l a s for the Fibonacci and Lucas Sequences
Division P r o p e r t i e s of Fibonacci N u m b e r s
G e n e r a l Fibonacci Sequences
The A s s o c i a t e d " L u c a s " Sequence
The Fibonacci Sequence and P a s c a l ' s Triangle
The Golden Section
M a t r i c e s and Fibonacci N u m b e r s
Continued F r a c t i o n s and Fibonacci N u m b e r s
This booklet should provide the m e a n s of becoming acquainted with
Fibonacci n u m b e r s and s o m e of t h e i r m a i n r a m i f i c a t i o n s . It should
s e r v e as a useful r e f e r e n c e for r e a d e r s of the Fibonacci Q u a r t e r l y who
wish to l e a r n about the m a i n a s p e c t s of Fibonacci n u m b e r s . It should
a l s o prove of value to groups of competent high school or college stud e n t s . While not r e c o m m e n d e d for the " p r o " , it might be a useful
r e f e r e n c e to have on hand to loan to students or fellow faculty m e m b e r s
who want to know something about Fibonacci n u m b e r s .
The booklet is now available for p u r c h a s e . Send all o r d e r s to:
B r o t h e r U. Alfred, Managing Editor, St. M a r y ' s College, Calif.
(Note. This a d d r e s s is sufficient, since St. M a r y ' s College is a post
office.)

A NEAR-GOLDEN RECTANGLE AND RELATED RECURSIVE SERIES


MARJORIE BICKNELL AND JAMES LEISSNER
Adrian C. Wilcox High School, Santa Clara, California
Thomas Jefferson High School, San Antonio, Texas

The r e c t a n g l e whose diagonals form e q u i l a t e r a l t r i a n g l e s with its


widths has s o m e s u r p r i s i n g p r o p e r t i e s , including a r e l a t e d F i b o n a c c i like s e r i e s of i n t e g e r s .
comparison,

Before d i s c u s s i n g this r e c t a n g l e , for l a t e r

we call to mind another r e c t a n g l e .

The famous

Golden

Rectangle h a s the p r o p e r t y that when a full-width s q u a r e is cut from


one end, the r e m a i n i n g p a r t has the s a m e p r o p o r t i o n s as the o r i g i n a l
rectangle,

the r a t i o of length to width being (1 + V5)/2.

Joseph Raab

d i s c u s s e d other golden-type r e c t a n g l e s [_lj , which have the p r o p e r t y


that when an i n t e g r a l n u m b e r

k of full-width s q u a r e s a r e cut from one

end, the r e m a i n i n g p a r t h a s the s a m e p r o p o r t i o n s a s the o r i g i n a l r e c t a n g l e . T h e s e golden-type r e c t a n g l e s a l s o have r e l a t e d s e r i e s of i n t e gers.


In the r e c t a n g l e whose diagonals form e q u i l a t e r a l t r i a n g l e s with
its widths, the r a t i o of length to width is

^ 3 , c e r t a i n l y not "golden."

But after cutting a full-width s q u a r e from one end, t h e r e a p p e a r s a


glitter a s the r a t i o of length to width b e c o m e s

(1 +

^)/2.

Operating

s i m i l a r l y on this r e c t a n g l e , the r a t i o b e c o m e s ^3 + 1, and r e p e a t i n g the


p r o c e s s one last t i m e m a k e s the r a t i o of length to width again ^ 3 .

2 - VJ

60

...,y, -.,..

V3 - 1

'

2fc|

2 - V3(e-2 V3 - 3
5-

S% -

1-jj

V3
227

228

A NEAR-GOLDEN RECTANGLE A N D RELATED

Oct.

Some m o r e " n e a r - g o l d e n " r e c t a n g l e s a p p e a r as m o r e g e n e r a l


c a s e s of r e m o v i n g s q u a r e s of the width in a r e c t a n g l e to obtain r e c tangles s i m i l a r to the o r i g i n a l .

To simplify the d i s c u s s i o n , we will

designate a r e c t a n g l e by a capital l e t t e r and its r a t i o of length to width


by the c o r r e s p o n d i n g s m a l l l e t t e r .
From a rectangle
the total n u m b e r
rectangle

P.

R with width x and length y + m x , r e m o v e

m of full-width s q u a r e s contained in R to obtain

From

P, remove

the total n u m b e r n of

s q u a r e s contained in P to form r e c t a n g l e

full-width

R' 0

y + mx

ky-^fe
*r

x - ny
*

ny

R!

JL

If R1 is s i m i l a r to R, then r 1 = r

so that y / ( x - ny) = (y + m x ) / x .

Solving for x / y 1 p, we find


I2~2
(mn + ^/m n + 4 m n ) / 2 n ,
/ 2 2
p = (mn + \*m n + 4 m n ) / 2 m ,
(Note that R:R' = rp, and that m = n = 1 yields the Golden R e c t a n g l e . )
When we cut full-width s q u a r e s from
number

P , if we r e m o v e an i n t e g r a l

n l e s s than the total n u m b e r of full-width s q u a r e s a v a i l a b l e ,

and if R1 and R a r e s i m i l a r ,
r = ( s/
^ (m+n)^ + 4 + m

i)A

p = ( V( m + n )2 + 4 + m

n)/2.

1965

RECURSIVE SERIES

(Note again the Golden Rectangle for

229

m = 1 and n = 0, when P = R ' . )

Suppose that we r e m o v e the full amount of available full-width


P and R', but R ! and R a r e not s i m i l a r .

s q u a r e s in forming
rectangle

T, s i m i l a r to R, can be obtained from

of an i n t e g r a l n u m b e r

R'

If a

by the r e m o v a l

q of s q u a r e s of the width of R', then

r = t = (M n 2 ( m + q ) 2 + 4n(m+q) + n(m - q))/2n,


p = ( ^ n 2 ( m + q ) 2 + 4n(m+q) + n(m + q))/2(m + q),
r 1 = ( V n 2 ( m + q ) 2 + 4n(m+q) + n(m + q ) / 2 n .
Again, q = 0 and m = 1 yields the Golden Rectangle, with
r = p = r f = (1 + >/5.)/2.

A l s o , q = m = n = 1 yields (for R and T) the

r e c t a n g l e with diagonals forming e q u i l a t e r a l t r i a n g l e s with its widths,


with p = (1 +

N/

3)/2.

The s i m i l a r i t y of form between the r a t i o (1 +


called 0 , and the golden r a t i o given above,

N3)/2,

hereafter

s u g g e s t s that we seek a

F i b o n a c c i - t y p e s e r i e s a s s o c i a t e d with p o w e r s of 0 .

Consider the fol-

lowing:

e --= (1
e2-. = (2

v/3)/2

= (1)0 + 0

^3)/2.

:= '(1)0 + 1 / 2

0 : = (5 + 3 >/3)/4 = (3/2)0 + 1/2

e 4 == (7

+ 4^3)/4

= (4/2)0 + 3 / 4

0 5 : = (19 + ll>^3)/8 = (11/4)0 + (4/4)


<? 6 == (26 + 15^3)/8 = (15/4)0 + (11/8).
The n u m e r a t o r s of e i t h e r the coefficients of

0 or the constant addends

and the coefficients of ^3 form the following s e r i e s :


15, 4 1 , 56, . . .

1, 1, 3, 4, 11,

It can be proved by induction that this s e r i e s is d e -

fined by
2n
2n+l

2n-l
2n

2n-2
2n-l

230

A N E A R - G O L D E N R E C T A N G L E AND R E L A T E D

where

P , = P ? = 1.

Oct.

A s e c o n d s e r i e s : 1, 2, 5, 7, 19, 2 6 , . . . ,

having

the s a m e r e c u r s i o n f o r m u l a s a s the above, a p p e a r s in the c o m p u t a t i o n


of p o w e r s of
If

0.

We s h a l l caLl t h e

0 = (1 + ^ 3 ) / 2 a n d 0 =

nth

(1 -

t e r m in the second s e r i e s

^ 3 ) / 2 , it i s n o t d i f f i c u l t to

R .
n
show

by i n d u c t i o n t h a t

Pn= (en -f1)/


R

n =

^-z^-n^

(0n+9Sn)/2 & - n / 2 3 ,

= l ,

w h e r e [x] i s t h e L a r g e s t i n t e g e r i n x .

2, 3,

....

The s e r i e s just defined b e a r a

s t r i k i n g r e s e m b l a n c e t o t h e F i b o n a c c i a n d L u c a s s e r i e s a s d e f i n e d by
t h e B i n e t f o r m u l a i n t e r m s of t h e g o l d e n r a t i o , w h e r e t h e
and

nth

nth

Fibonacci

L u c a s n u m b e r a r e g i v e n r e s p e c t i v e l y by

rp
an - pn
n , Rn ,
1 + ^5
1 - ^S
a
F =
- , L - a + 6 for a =
, p =
J5
n
2
2
U s e of t h e a b o v e f o r m f o r P

and R
n

and s t a n d a r d limit t h e o r e m s
n

l e a d s to
Limit

P0 /P~
, =
2n' 2 n - l

and

Limit

R0 / R 0
, = d> ;
r
2n' 2 n - l

n-> co
n>co
L i m i t P 0 M / P , =2(9 a n d L i m i t R_ , . / R ~ = 2 0 .
2 n + l ' 2n
2n+l'
2n
n-a* 0 0
n __^co
F i n a l l y3 , a s n i n c r e a s e s , R / P
oscillates about its limit, N / 3 .
n' n
A l s o e s t a b l i s h e d by i n d u c t i o n a r e f o r m s f o r p o w e r s of #0
0n

= {P
X

0 ) / 2

Bn-D/2]

+ p

^' n

/ 2

[n/2]

+ p

n-1'

[(n+l)/2]

"

and
r

= ( - 2 ) n ( P n + 1 / 2 t n / 2 ] - P n 0 / 2 D11"1)/2] )

For comparison,

if

IjtJl
?

where

a,

then

a*1 = ( L

+ F

is the nth F i b o n a c c i n u m b e r and L

n
n

\f5)/2
'

the nth Lucas n u m b e r .

1965

RECURSIVE SERIES

231

Other t h e o r e m s , a l s o possible to e s t a b l i s h by induction, a r e :


2n
1 P . = P ? ,, - x( P 9 . + l ) / 2 ,
. ,
I
2n+l
2n-l
''
i=l

2n+l
1=1

2(2n-l)
v
.""%

P = P P
- p
i
2n 2n+l
2n-1

1=1

P P , , - P n P , ? = (~l)n + 1 .
n n+3
n+1 n+2
Considering the even o r d e r e d e l e m e n t s and the odd o r d e r e d e l e m e n t s of the s e r i e s s e p a r a t e l y leads to
P

= 4P

2n

p
^2n+l

2n-2 " P2n-4


4p
p
^2n-l
^2n-3'

which in t u r n can be used to p r o v e the following r e l a t i o n s h i p s between


R

and p , and s u m m a t i o n f o r m u l a s for even or odd e l e m e n t s of the


n
n
s e r i e s P, :
n
R9 = P 9 , + P 9 ,
2n
2n-l
2n
Zn

Zn-1

LTI

n
. ,, - l )"/ 2 = x( 3 P2n
- P 92n-29 - l ); // 2 ,
9
.2 . P 92i. = x( P 2n+l

i=l
and

n
V
^

i=l

P 92,i - l1 = P 92n = ( P 92n+3


,~ - P 92 nx+l l) '// 2 .

REFERENCES
Joseph A. Raab, "A G e n e r a l i z a t i o n of the Connection Between the
Fibonacci Sequence and P a s c a l ' s T r i a n g l e , " Fibonacci Q u a r t e r l y ,
3:1, Oct. , 1963, pp 8 21-32.
xxxxxxxxxxxxxxx

FACTORIZATION OF 36 FIBONACCI NUMBERS F n WITH n>100


L. A. G. DRESEL AND D. E. DAYKIN
Reading University,

The Fibonacci numbers F

England

are defined by F, = F>= 1 and

J
n
1
2
F ,, = F + F , for n > 1. W e present be Low the factorization of
n+1
n
n-1
^
certain F
with n > 100. The factors of F
before the double
n
n
asterisk are improper factors of F
(they divide F
with m < n),

and those behind the double asterisk are proper factors of F . All the
factors shown are believed to be primes. W e obtained the results on
the Elliott 80^-computer at Reading University, and we hope to discuss
our methods and extend the table in a later paper.
F102 = 2 3 * 1597 * 3571 * 6376021 ** 919 * 3469
F104 = 3 * 7 * 233 * 521 * 90481 ** 103 * 102193207
F105 = 2 * 5 * 13 * 61 * 421 * 141961 ** 8288823481
F106= 953 * 55945741 ** 119218851371
F108 = 2 4 * 3 4 * 17 * 19 * 53 * 107 * 109 * 5779 ** 11128427
F110 = 5 * ll 2 * 89 * 199 * 661 * 474541 ** 331 * 39161
F112 = 3 * 7 2 * 13 * 29 * 47 * 281 * 14503 ** 10745088481
F114 - 2 3 * 37 * 113 * 797 * 9349 * 54833 ** 229 * 95419
F116 = 3 * 59 * 19489 * 514229 ** 347 * 1270083883
F117 = 2 * 1 7 * 233 * 135721 ** 29717 * 39589685693
F118 = 353 * 2710260697 ** 709 * 8969 * 336419
F120 = 2 5 * 3 2 * 5 * 7 * 11 * 23 * 31 * 41 * 61 * 2161 * 2521 ** 241 * 20641
F126 = 2 3 * 13 * 17 * 19 * 29 * 211 * 421 * 35239681 ** 1009 * 31249
F128 = 3 * 7 * 47 * 1087 * 2207 * 4481 ** 127 * 186812208641
F129 = 2 * 433494437 ** 257 * 5417 * 8513 * 39639893
F130 = 5 * 11 * 233 * 521 * 147 36206161 ** 131 * 2081 * 24571
F132 = 2 4 * 3 2 * 43 * 89 * 199 * 307 * 9901 * 19801 ** 261399601
F134 = 269 * 116849 * 1429913 ** 4021 * 24994118449
F138 = 2 3 * 137 * 139 * 461 * 829 * 18077 * 28657 ** 691 * 1485571
F140 = 3 * 5 * 11 * 13 * 29 * 41 * 71 * 281 * 911 * 141961 ** 12317523121
F1.44 = 2 6 * 3 3 * 7 * 17 * 19 * 23 * 47 * 107 * 1103 * 103681 ** 10749957121
F147 = 2 * 13 * 97 * 421 * 6168709 ** 293 * 3529 * 347502052673
F148 = 3 * 73 * 149 * 2221 * 54018521 ** 11987 * 81143477963
232

1965

FACTORIZATION O F 36 FIBONACCI NUMBERS F

233

F150 = 2 J * 52
-Ml * 31 * 61 * 101 * 151 * 3001 * 230686501 ** 12301*18451
F156 = 2 4 > 3 2 * 79 * 233 * 521 * 859 * 90481 * 135721 .** 12280217041
F162 = 2 3 * 17 * 19 * 53 * 109 * 2269 * 4373 * 5779 * 19441 ** 3079- 62650261
F165 = 2 * 5 * 61 * 89 * 661 * 19801 * 474541 ** 86 461 * 518101 * 900241
F168 = 2 5 * 3 2 * 7 2 * 13 * 23 '* 29 * 83 * 211 * 281 * 421 * 1427 * 14503
** 167 * 65740583
F174 = 2 3 * 59 * 173 * 19489 * 514229 * 3821263937 ** 349 * 947104099
F180 = 2 4 * 3 3 * 5 * 1 1 * 1 7 * 1 9 * 31 * 41 * 61 * 107 * 181 * 541 * 2521
* 109441 ** 10783342081
F190 = 5 * 11 * 37 * 113 * 7 61 * 9349 * 29641 * 677 35001 ** 191 * 41611
* 87382901
F198 = 2 3 * 17 * 19 * 89 * 197 * 199 * 9901 * 19801 * 18546805133 ** 991
* 217 9 * 1513909
F204 = 2 4 * 3 2 * 67 * 919 * 1597 * 3469 * 3571 * 63443 * 6376021 ** 409
* 66265118449
F210 = 2 3 * 5*11 * 13 * 29 * 31 * 61 * 71 * 211 * 421 * 911 * 141961 *
8288823481 ** 21211 * 767131
F216 = 2 5 * 3 4 * 7 * 1 7 * 19 * 23 * 53 * 107 * 109 * 5779 * 103681
* 11128427 ** 6263 * 177962167367
F228 = 2 4 * 3 2 * 37 * 113 * 229 * 797 * 9349 * 54833 * 95419 * 29134601
** 227 * 26449 * 212067587

xxxxxxxxxxxxxxxxx
LETTER TO THE EDITOR
ERIC HALSEY
Redlands, California

Re: My a r t i c l e The F i b o n a c c i Number F w h e r e u i s not an i n t e g e r


in is sue n u m b e r 2 of the c u r r e n t volume of the Q u a r t e r l y . I have d i s c o v e r e d t h a t , due to e x c e s s i v e h a s t e and timidity on m y p a r t , I placed
undue r e s t r i c t i o n s on the l e t t e r u. This v a r i a b l e can a s s u m e not only
all r a t i o n a l v a l u e s , a s stated in the a r t i c l e , but a l l r e a l values a s well.
Obviously, only for r a t i o n a l values c a n a complete n u m e r i c a l e x p r e s sion of F be obtained.
u

COMMENTS ON " T H E GENERATED, COMPOSITIONS


YIELD FIBONACCI NUMBERS"
HENRY WINTHROP
University of South Florida, Tampa, Florida

The following explanations will serve to round out the paper mentioned above which appears in T h ^ F i b o n a ^

3, No. 2,

131-4).
The expression

F(h., <fi) of model display (1) designates the par-

titions of the integer, i, in which the partitions are expressed as functions in <p and in which the coefficient of each partition represents the
number of possible permutations of that partition.
The general term of model display (3) can be given as
= ik + [(i-1) + 2 ]!k 2 + [(i-2) + 4] Ik 3 + . . .

(1)

1i-2>J3!

(i-3)!5!

+ [i-(n-l) + 2(n-l)] I k n + . . . +
k1
irr n )!(2n-l): ~~
(i-n)!(2n-l)l
where the coefficient of k

is the n-th term of the 2r-th order of the

figurate numbers.
A discussion of figurate numbers of various orders will be found
in Higher Algebra by Hall and Knight (Macmillan, 1936, 4th edition),
pp.

319-22.
The following additional references to the paper in question will

be found of value by the reader.


1.

H. Winthrop,

"Mathematics In The Social Sciences,"

School

Science and Mathematics, 1957, Vol. 57, pp. 9-16.


2.

H. Winthrop, "On The Use of Difference Equations In Behavioral


Diffusion Theory, " School Science and Mathematics, 1958, Vol.
58,

30

pp. 1-6.

H. Winthrop,

"A Kinetic Theory Of Socio-Psychological Dif-

fusion, " Journal of Social Psychology,


4.

H. Winthrop,

1945, Vol. 22, 31-60.

"Experimental Results In Relations To A Mathe-

matical Theory Of Behavioral Diffusion, " Journal of Social Psychology, 1958, Vol. 47, 85-99.
Continued on page 240.
234

ELEMENTARY PROBLEMS AND SOLUTIONS


Edited by A. P. HILLMAN
University of Santa Clara, Santa Clara, California

Send all c o m m u n i c a t i o n s
Solutions to P r o f e s s o r

r e g a r d i n g E l e m e n t a r y P r o b l e m s and

A. P . Hillman,

D e p a r t m e n t of

Mathematics

a n d S t a t i s t i c s , U n i v e r s i t y of N e w M e x i c o , A l b u q u e r q u e ,

New M e x i c o .

E a c h p r o b l e m or solution should be s u b m i t t e d in legible f o r m ,

prefer-

a b l y typed in double s p a c i n g , on a s e p a r a t e s h e e t o r s h e e t s in t h e f o r mat used below.

Solutions

s h o u l d b e r e c e i v e d w i t h i n t w o m o n t h s of

publication.
B-70

Proposed by Douglas Lind, University

Denote
taining

B-71

Show that the following e x p r e s s i o n ,

con-

e x ( J ex( J e x ( . . . / ex( f x d x ) d x ) . . . d x ) d x ) d x
^0
JO
JO
JO

,, / F , _, w h e r e
n+1 '
n+2

is the

a"

+ a~

n-th

Fibonacci number.

Proposed by Douglas Lind, University

Find
B-72

bbyy e x ( a ) .

Va.

integrals,

I
^0
equals

of Virginia, Charlottesville,

of Virginia, Charlottesville,

+ a - 4 +. . . , w h e r e

Proposed by ]. A. H. Hunter, Toronto,

a = (1 +

Va.

5)/2.

Canada

E a c h d i s t i n c t l e t t e r in this s i m p l e a l p h a m e t i c stands for a p a r t i c u l a r and different


Fibonacci series,

digit.

We a l l k n o w h o w r a b b i t s

link up with the

so now evaluate our R A B B I T S .


R A B B I T S
BEA R
R A B B I T S
A S
A

S E R I E S

B - 7 3 Proposed by Douglas Lind, University of Virginia, Charlottesville,

Va.

P r o v e that
n

2
k=0

2 (?) (k+TTl) = l +
j=0

2n+r-2

2
m=0

235

2 (mTX) .

p=0

236
where

ELEMENTARY PROBLEMS AND SOLUTIONS

Oct.

( ) = 0 for n < r .
V

B - 7 4 Proposed by M. N. S. Swamy, University of Saskatchewan,

Regina,

Canada

The F i b o n a c c i polynomial f (x) is defined by 1 = 1, f? = x,


and f (x) = xf . (x) + f _(x) for n > 2. Show the following:
n
n-1
n-2n
(a)
x 2
f (x) = f ,. + f -1 .
N
'
, r
n+1
n
r=l
(b)
f , ,. = f , 1 f ,. + f f .
*
m+n+1
m+1 n+1
m n

(c)

f (x)

S
j=0

(n

ZJ l

w h e r e [kj is the g r e a t e s t i n t e g e r not exceeding k.

Hence show that

the n - t h F i b o n a c c i n u m b e r
F

O-D/2]
s
i=o

.j_i
(nn ! ) .
J

B - 7 5 Proposed by M. N. S. Swamy, University of Saskatchewan,

Regina,

Canada

Let f (x) be a s defined in B - 7 4 . Show that the d e r i v a t i v e


n
,
n-1
f n x(x)' = X, fr x(x)' fn - r x (x)' for n > 1.
r=l
SOLUTIONS
ONE, TWO, THREE OUT
B-58

Proposed by Sidney Kravitz, Dover, New

]ersey

Show that no F i b o n a c c i n u m b e r other than 1, 2, or 3 is equal to


a Lucas n u m b e r .
Solution by Douglas Lind, University of Virginia, Charlottesville,

Since
(I)
{L)

L, = F,

Va.

, + F, ,, , the a s s e r t i o n i s equivalent to
F
n

= F
+ F
*k-l
k+1*

1965

ELEMENTARY PROBLEMS AND SOLUTIONS

If k > 3, the

237

n > k+1 and (1) is clearly impossible since


F

k-1
Impossibility for

XI

<

k+1

_L1

k+1

XO

k+2

k ^ 3 implies impossibility for

signs a r e different.

For

-3 < k < 3 we find

k i -3
= L

&

F~ = L = 2, F , = L, = 1, and
0, 1, and 2 respectively.

since only

= 1,
l

F . = L~ = 3, corresponding to k = - 1 ,

Hence these a r e the only solutions.

(The crux of this problem is solved in the discussion of equation (12)


in C a r l i t z ' "A Note on Fibonacci Numbers,
No.

this Quarterly 1 (1964)

2 pp. 15-28).

Also solved by J. L. Brown, Jr.; Gary C. McDonald; C. B. A. Peck; and the


proposer.
B-59

Proposed by Brother U. Alfred, St. Mary's College,

California

Show that the volume of a truncated right c i r c u l a r cone of slant


height
&

with

,
n-1

and

in
n+1

^"<Fn+l

the d i a m e t e r s of the bases is

n-l)/24.

Solution by Douglas Lind, University of Virginia, Charlottesville,

Va.

It is we 11-known that if h is the height of the frustrum of a right


c i r c u l a r cone, s the slant height,
b a s e s , then the volume

and

and

the radii of the

V is

V = (rrh/3)(rj + r } r 2 + r^)
= (rr/3) \ / 7 ^ ( r 2 - r 1 ) 2 ( r J + r ^
For this rproblem

r, = F . / 2 , r 0 = F , , / 2
1
n-1'
2
n+1'

+ r2).
and

s = F ,
n

so that

V = 5 \v / F 2 - x(F , , - F T ) 2 / 4 x ( F 2
. + F . F , 1 +F2
)/4
3
n
n+1
n-1' '
n-1
n-1 n+1
n+1"
77 V F 2 - F 2 / 4 x( F 2 1 +F
.F x l + F 2
)/l2
n
n'
n-1
n-1 n+1
n+1"
\/377 F

( F 2 . + F . F ,_ + F 2 , 1 ) / 2 4
n-1
n-1 n+1
n+1"

238

ELEMENTARY PROBLEMS AND SOLUTIONS

Oct.

- V I , ( F n + 1 - F n . 1 ) ( F ^ 1 + F n _ l F n + 1 + F^ + 1 )/24

"

n-1>/24'

We remark that the area

A of the curved surface of the frustrum is

A = n F (F , . + F
, )/2 = {n/2)F L .
n n+1
n-1"
'
n n
Also solved by Carole Bania, Gary C. McDonald, Kenneth E. Newcomer,
C. B. A. Peck, M. N. S. Swamy, Howard L. Walton, John Wessner, Charles
Ziegenfus, and the proposer. McDonald also added the formula for the
curved surface.
B-oO

Proposed by Verner E. Hoggatt, Jr., San Jose State College, San Jose,

California

n-th

2
L^ L~ x o + 5 F 0 , 1 = 1 , where F
and L
are the
2n 2n+2
2n+l
n
n
Fibonacci number and Lucas number, respectively.
Show that

Solution by 2nd Lt. Charles R. Wall, U. S. Army, A. P. 0., San Francisco,

Calif.

Using my second answer to B-22 (Vol. 2, No. 1, p. 78),

2(n+1)

2n

= 5F

(n+l)+n

= 5F2n+l

= 5F

(n+l)-n

Ll+1

Thus
2
L

2n+2 L 2n "

5F

2n+l

= l

'

Also solved by J. L. Brown, Jr.; J. A. H. Hunter; Douglas Lind, Kathleen Marafino,


Gary C. McDonald, C. B. A. Peck, Benjamin Sharpe, M. N. S. Swamy, Howard L.
Walton, John Wessner, Kathleen M. Wickett, David Zeitlin, Charles Ziegenfus, and
the proposer. Also by David KLarner.
MODULO THREE
B-6l

Proposed by J. A. H. Hunter, Toronto, Ontario


Define a sequence

U, , U ? , . . . by

U = U
,+n
n
n-1
Prove that

= 0(mod n) if

Solution by John Wessner, Melbourne,

+n +l

U, = 3
for

$ 0 (mod 3).

Florida

and

n > 1 .

1965

ELEMENTARY PROBLEMS AND SOLUTIONS

239

An a l t e r n a t i v e r e p r e s e n t a t i o n for U is
n
n
7
Un=
1
(k^+k+1).
k=l
Upon expanding the individual s u m s involved we obtain
Un=

[n(2n+l)(n+l)/6]

Hence,

+ [n(n+l)/2]

+ n = (n/3)

[(n+2)(n+l)+3] .

U = 0(mod n) if and only if (n+l)(n+2) = 0(mod 3). This

dition obtains if and only if n

con-

^ 0(mod 3).

Also solved by Robert J. Hursey, Jr., Douglas Lind, Gary C. McDonald, Robert
McGee, C. B. A. Peck, Charles R. Wall, David Zeitlin, and the proposer.

UNIQUE SUM OF SQUARES


B-62

Proposed by Brother U. Alfred, St. Mary's College,

California

P r o v e that a Fibonacci n u m b e r with odd s u b s c r i p t cannot be r e p r e s e n t e d a s the sum of s q u a r e s of two Fibonacci n u m b e r s in m o r e than
one way.
Solution by J. L. Brown, jr., Pennsylvania

State University,

State College,

Pa.

F r o m the identity3
F n .,
2n+l
F . .,
2n+l
k > n

F 0 ,, = F + F, ,, , (n> 1) it follows
2n+l
n
n+1
< (F + F , , ) = F
T h e r e f o r e , any r e p r e s e n t a t i o n
x
n+2
n
n+1'
+?.
= F, + F 2 x(k < m) m u s t have both k and m < n+1.
k
m
'
(otherwise
Ff + F Z < F ^ + F z ^ , = F , ,. for k > 2).
1
k
rn
n
n+1
2n+l
'

that

2
2

Also solved by Douglas Lind, Joseph A. Orjechouski


C. B. A. Peck, and the proposer.
AN ISOSCELES

and Robert McGee

(jointly),

TRIANGLE

1 3 - 6 3 An old problem whose source is unknown, suggested


New Jersey.

In

by Sidney Kravitz,

A ABC let s i d e s AB and AC be equal.

point D on side AB such that AD = CD = BC.

Dover,

Let t h e r e be

Show that

2cos $ A = A B / B C = (1 + \/5)/2 ,
the golden m e a n .
Solution by John Wessner, Melbourne,

Then

Florida

By i n s p e c t i o n of the figure and the law of c o s i n e s


AD 2 = C D 2 + A C 2 - 2CD* AC cos f A.

240
Since

ELEMENTARY PROBLEMS AND SOLUTIONS


AD = CD = BC

and

AB = AC,

Oct.

it follows i m m e d i a t e l y that

2 cos $ A = A C / C D = A B / B C .
The second r e s u l t c o m e s from the fact that
$ B = $ BDC = $ A + $ DCA = 2 $ A
and hence
$ A = 36 and 2 cos A = (1 + >/5)/2.
(See N. N. Vorobyov: The F i b o n a c c i N u m b e r s (New York, (1961) p . 5 6 . )
Also solved by Herta Taussig Freitag, Cheryl Hendrix, Kathleen Marafino, and
Carol Barrington (jointly), ]. A. H. Hunter, Douglas Lind, James Leissner, C. B. A.
Peck, Kathleen M. Wickett, and the proposer.

XXXXXXXXXXXXXXX

Continued from page 234.

5.

H. Winthrop,

"The M a t h e m a t i c s Of The Round Robin, " M a t h e -

m a t i c s Magazine (In P r e s s ) .
6.

H. Winthrop,

"A M a t h e m a t i c a l Model F o r The Study Of The

P r o p a g a t i o n Of Novel Social Behavior, " Indian Sociological Bulletin, July 1965, Vol. II. (In P r e s s )
7.

H. Winthrop, "Some G e n e r a l i z a t i o n s Of The Dying Rabbit P r o b lem, " (In P r e p a r a t i o n ) .

8.

N. N. Vorob'ev, Fibonacci N u m b e r s , B l a i s d e l l Publishing Company, New York, 1961.


XXXXXXXXXXXXX

ASSOCIATION PUBLISHES BOOKLET


B r o t h e r U. Alfred h a s just completed a new booklet entitled: I n t r o d u c tion to Fibonacci D i s c o v e r y . This booklet for t e a c h e r s , r e s e a r c h e r s ,
and bright students can be s e c u r e d for $1.50 each or 4 copies for
$5.00 from B r o t h e r U. Alfred, St. M a r y ' s College, Calif.

Вам также может понравиться